Sunteți pe pagina 1din 115

WAR​N​ING !!!!!!

PLEASE BE AWARE THESE ANSWERS MIGHT NOT BE 100% CORRECT , I SOLVED IT IN


THE WAY I THOUGHT ITS CORRECT , SO PLEASE CONFIRM ALL ANSWERS YOURSELF.

1 A Dentist did pulpotomy of primary molar and then after few moths he did stainless steel
crown. What band he gets to claim.
Band 1
Band 2
Band 3
Band 4
No Claim

2 After doing mesio-angular impaction of lower third molar, which suture material is
best used to close mucoperiosteal flap
3-0 Silk cutting needle
3-0 Vicryl cutting needle
Catgut

3 Patient has TIA (Transient ischemic attack three months ago, you have planned
extraction and patient is on aspirin. What is the best way to proceed?
Delay the extraction for three months
Go ahead with extraction, following appropriate local measures
Refer to Oral surgery
Stop aspiri​n

4 Patient has an over jet of above 9mm which UDA band it will be
Band 1
Band 2
Band 3
Band 4
Band 5

5 Over jet of 4.5 mm criteria which UDA band it will be


Band 1
Band 2
Band 3
Band 4
Band 5
only score 4 and 5 iotn will be nhs treatment

6 Autoclave temperatures (at what temperature the sterilization is achieved)


121 degrees centigrade for 15 minutes
121 degrees centigrade for 3 minutes
131 degrees centigrade for 15 minutes

p 354 pink book

7 A patient with irritable bowel syndrome what do you see.


Mucosal blistering
Mucosal tags
Herpitiform ulcers
n irritable bowl syndrome u find herpetiform ulcer
In irritable bowel disease like crohns disease u find mucosal tags

8 Patient had pain in the chest after prolonged dental session, with an extraction procedure,
what is the best medicine to give
GTN
ASPIRIN

9 Patient is insulin dependent diabetic, complains of faint after prolonged dental


session, what is best to give
Oral Glucose
IV Glucagon
Insulin
1M Glucagon

pink p 547 tx of hypoglycaemia:

Treatment

If conscious, give glucose orally in any available form. Repeat if necessary every 10–15min.
Get help.
If impaired consciousness, give buccal glucose gel and/or1mg of IM glucagon.
If unconscious, protect airway, place in recovery position. If the correct kit is available, establish
IV access and give up to 50mL of 20–50% glucose.
10 Patient is waiting in waiting room and collapse and faints. Patient is cold, clammy, but pulse
is good. What do you give?
IM Glucagon
Oral Glucose
GTN
Hydrocortisone
Diazepam

csully 7ed ​Table 1.3 ​Common emergencies

11 Patient is suffering myocardial infarction in dental chair, which position is best for patient
Upright sitting position
Laying flat
Recovery position
Lay the pt flat and raising legs

pink p 537

12 A 90-year old gentleman presented to clinic who is edentulous and has dentures upper and
lower full 15 years old. Denture bit uncomfortable, tooth structures little bit worn out, freeway
space 2-4mm, polished surfaces satisfactory and occlusal wear
minimal. How would you proceed?
Copy dentures
Hard reline
Soft reline
Construction of new dentures

13 When do not extract impacted 8's


Extraction of 8's recommended by orthodontist
Extraction of 8's recommended in patient undergoing renal transplant
-immunosuppresent
47years old with moderate periodontitis
Second permanent molar undergoing internal resorption due to third molar

14 Non dental origin, No clinical or radiological findings. Patient complains of pain maxillary
sinus and ear deafness, unilateral ulcer on face and forehead
Herpes Simplex
Herpes Zoster
Paramyxo virus
Orthomyxo virus
Ep-stein bar virus

15 How do you check maxillary plane


Fox-bite-plane
OCcluding the upper and lower denture

16 What sort of matrix is best for restoring disto-occlusal restoration of 7?


Sectional matrix to protect neighbouring tooth
Auto matrix
Tofflomeir matrix to apply material

17 How do you get incisal guidance for setting incisal teeth?


Study casts
Unsupported/supported labial fullness

18 Burning mouth syndrome which part of the oral cavity is more painful
Tongue

Lichenoid reaction seen associated with


Atenolol
Ramipril

19 GTN in Angina
Relaxes heart muscle
Decreases blood Pressure
Increase positive ionotropic charge
Dilation of arterioles and venules

20 had trauma - Nasal- midface area and has watery discharge and blood discharge from nose
Orbital fracture
Zygomatic fracture
Lefort-l
Lefort-ll

21. Patient with only mid-root fracture of front tooth, what is apropriate splinting time?
1 week
2 week
​4 week
3 months

pink p 102

22 Patient with lower four central incisors with alveolar fracture, you splinted and immobilized
successfully, and then what is the next thing to do.
Soft diet and review

23 Patient had trauma with minimal mobility of front teeth, no symptoms or pain or anything,
what is the best approach
Splinting one week
Splinting two weeks
Splinting 4 weeks
Splinting 6-8 weeks
Soft diet and review​ ~

24 Patient had trauma 8 days ago, upper central incisor palatal luxation mild, not interfering with
occlusion but tooth non vital.
What do you do?
Pulp extirpation +CaoH2 dressing
Reposition
Reposition +Splinting ~

25 Best material for using vital pulpotomy


.​.,Ferric sulphate
Formocresol

26. Patient had trauma on 14 (vital pulp), only thin buccal cusp remaining, best treatment option
MOD-Onlay
inlay
Direct composite MODL
Extract and options for replacement

27.18 months of age, what is fluoride supplement for child living in 0.25 ppm fluoridated water?
0.25mg
0.5 mg
1 mg
None

28 Six-year-old - Non fluoridated water, what is the fluoride supplement dose


0.25 mg
0.5 mg
1 mg

​ .6ppm​, what is the fluoride supplement dose


29 Six-year-old child with fluoridated water of 0
given
0.25 mg
0.5 mg
1 mg
Note the dose of the fluoridated water

30 3-year old with fluoride concentration of 0.3 ppm fluoridated water, what is the fluoride
supplement dose given
0.25 mg
0.5mg

31 Duraphat in high caries risk group children is applied every


2 months

4monttls​ ~
6 months

8 months
One-yearly

32 Reason for failure of eruption of lower permanent molar


Canine palatally place
Crowding in premolar area
Failure of eruption of central incisor
Absence of showing lateral incisor






33 Failure of normal path of eruption
Abnormal position of crypt
Early loss of deciduous teeth
Supernumerary tooth

34 Cervical line and dark lines on Central incisors, molars and incisal edges, most possible
reason could be
Ameologensis
Dentinogenesis
​ Childhood illness
Hypoplasia

35 An edentulous patient presents with ulcer in retro molar pad area first line of management
would be
Adjusting the denture
Incisional biopsy
Periapical xray

36 How should be the floor of dental surgery?


impervious
Absorbable

37:​Possible infection that would occur from needle stick​ injury in a person who is appropriately
or properly immunized
a-Hepatitis B
b-Hepatitis C
c-Hepatitis A
d-HIV
​30% chances for HBV incase of ​non immunised person exposed to infected blood, 3% HCV and 0.3% HIV

39 Got a call from mother of a 3-year old child, who had trauma and avulsed the front tooth.
What instruction you will give to
Attend Surgery as soon as possible with no special consideration to the tooth
Place tooth in milk and attend surgery as soon as possible
Place in the salt water and attend surgery as soon as possible
Place the tooth in its position back and attend surgery as soon as possible

40 A 3-year-old boy avulsed 61, and mother re-implanted it in its socket and clot is holding the
primary tooth. IOPA shows permanent tooth present, what would you do
Leave it and review
Splinting for one week
​Remove and space maintenance

42 Patient has painful symptoms at the time of eating and also has ear pain, what x-ray would
you consider. (The option of occlusal x-rays was not given in the choices)
OPG
Oblique Lateral -
Lateral chepalograph
Postero-anterior view

41 A child has grossly broken down upper and lower molars, carious tooth, which x-ray would
you consider
IOPA
Bitewings_
Vertical Bitewings
DPT

43 A dental practitioner can be


1-Radiologyadviser+physicst+operator
2-Superviso r + adviser+physicst
3-practioner+refer+operator

4-Practioner + physicst +supervisor


5-Physicst + supervisor + 0perator
44 Risk of cancer in dental Radiography periapical
● 1:20,000
● 1:200,000
● 1:2,000,000 in essential
● 1:20,000,000 in modern
45 Which cells are affected by Bisphosphonates?
Mesencymal cells

Undifferentiated cells
Oesteoclasts

pink page596

49 When preparing the Apical Zone, the use of the files sequentially from apex to backwards
(lower the size of instrument) - what is the best distance to achieve good apical area
preparation
0.5mm
1.0mm
l.5mm
2.5mm
3.5mm

51 Best material to use in the root caries and buccal caries in elderly patient.
Hybrid Composite
Amalgam
​Silicate cement
Zinc phosphate

47 A diabetic patient complains of pain on a root canal treated tooth. Root canal is done 2 years
ago. On examination the restoration on the tooth seems leaking. Reasons for pain and failure
No proper obturation
Medical condition
Lack of coronal seal
Inadequate biomechanical preparation
48 Best success of root canal is achieved by
Straight canals
Straight line access
Widening of aperture of root canals
Proper irrigation all the times under presence of rubber dam

50 Prolonged use of corticosteroids in adults may result in or can cause


Alopecia
Insulin - diabetes
Libidos
Thinning of skin
Less arousal
If its topical steroids-then thinning of skin
If systemic steroids-then drug induced hyperglycemic state (diabetes)

52 A child patient is brought to the dental clinic, parents complain of child having trauma and
has fallen down, hurting his teeth. You suspect NAI's. Which is the most appropriate finding
which would give suspension of NAI'S?
Red eye
Neck bruise
Child agitated and distressed
Injuries to shin and knee areas

53 What is the bpe score if gingival recession of 2mm and band completely disappears
1
2
3
4
4* due loss of attachment >7

54 What is the bpe score if Gingival recession 2mm +5.5mm


1
2
3
4

4* due loss of attachment >7


55 On examination of four teeth in a sexant except for L4 has no bleeding on probing. What is
the BPE SCORE?
O
1
2
3
4

For a sextant to qualify for recording, it must contain at least 2 teeth. (If only 1 tooth is

present in a sextant, the score for that tooth is included in the recording for the adjoining

sextant).
bpe advice sheet

56 Patient has damaged infra-orbital nerve - orbital floor fracture, patient presents to you after
two hours later. What symptoms
you would find, choose most appropriate one

Peri-orbital edema
Loss of visual acuity
Loss of sensation or paresthesia over cheek

Loss of sensation and paresthesia over cheek.


Edema takes 2-3 days to develop.

emq 57 what you would do


1-Dismissal
2-Grievance procedure
3-Informal discussion
4-Suspension and investigatin
5-Written warning

Options are
a-Dental assistant want to explore future carrier opportunitie​ > 3 informed discution
c-Dental assistant suspected of the theft​> 4-Suspension and investigatin

d-Repeated complaints of letters from patients regarding dental assistant ​> 5-Written warning

e-Dental staff or person failure to respond repeatedly even after giving many​>1-Dismissal

​ 2-Grievance
f-A staff member repeatedly re-heard many times comments from colieagues >
procedure

58 For retention of post which is not important


Diameter
Size
Luting cement
Type of post

59 RCT treated tooth, post placed, which is the best option to restore the tooth. Nayyar
technique used, good tooth structure 50%
Gold crown
Gold inlay
PJC
Gold Onlay

60 The movement of tongue on protrusion is affected to lateral side -which nerve is involved.
Patient recently had surgery and radiation close to submandibular gland
Lingual
Glossopharyngeal
Hypoglossal
Chordatympani

pink p 524
63 Chose the correct option
1-General anesthetics and day surgery
2-General anesthetics and one day inpatient
3-Local anesthetics and oral sedation
4-Local anesthetics and inhalation sedation
5-LocaI anesthetics
Options:
a-Child 3-years-old, removal of all deciduous primary molars >​ 1
b-12- years-old very anxious patient, removal of all 5's >​ 4
c-Extraction of fully erupted molar in 4-month-old pregnant lady​>5
d-Child for extractions who is medically compromised > ​ 2

1 GA day surgery
4 LA inhalation
5 local anesthesia
2 GA one day inpatient

61 When patient complaints about a treatment and unhappy about the procedure and gives a
complaint, in how many days you have to acknowledge the complaint
1 day
2 days
3 days
10 days
20 days

pink p670
Acknowledge receipt/respond to the complaint within 3 working days

of receipt​3 ​(2 days for NHS complaint in Wales​4​).

62 Patient has cardiac arrhythmias on warffarin and yesterday had INR of 3, today needs
extraction. What do you do?
Differ the extractions
Consult phvsician
Alter the dose of warfarin
Do the extraction and control post operative bleeding
pink p596
Patients requiring minor dental procedures (including extractions) who have an INR below 4 may continue
warfarin without dose adjustment’. INR should be checked within 72 hours of the procedure starting. If
multiple extractions are required, then a single extraction should be carried out first. Subsequent extractions
of two to three teeth at a time may be carried out if recovery is uneventfu​l.

​ lose fit tray with hyper gap reflex. Which


64 Secondary impressions for edentulous patient c
material you use for taking an impression
Alginate
Silicone rubber based material
Plaster of Paris
Zinc oxide eugenol

master p130
Requirements for trays

Upper trays. Spaced (two thickness of modelling wax) light-cured trays should be prescribed for use with
silicone or alginate impression material. ​Lower trays. Close-fitting light-cured trays for use with zinc
oxide/eugenol paste can be requested.

65 Palatally impacted upper maxillary canines. How would you know? Whether they are
impacted palatally or buccally and which of the following x-rays are best option
Paralleling technique
Bisecting technique
Bitewings
OPG
Taking two x-rays, with paralleling technique
it shd be parallex

66 Defective horizontal angulations while taking bitewings results in?


Overlapping of contact points
Dark radiograph
Light radiograph

eric

● ​Failure to align the X-ray tubehead correctly in the horizontal plane, either
– Too far posteriorly (​coning off or ​cone cutting)

– Too far anteriorly — rarely (​coning off or ​cone cutting)

– Not aimed through the contact areas at right angles to the line of the arch and the image receptor causing
overlapping of the contact areas.

67 Which of the following antifungal medication interact with warfarin and enhances its effect?
Ketacanazole
Micanazole​_
Flucanazole systemic antifungal
Amphotrocin B

68 A patient came after two days of extraction, complains he had pain for two days, examination
showed localized swelling, no
lymphadenopathy. What is the first line of treatment?
Possible dry socket -irrigation and placement of sedative medicament
Irrigation - curettage of socket, antibiotics and analgesics
Irrigation and antibiotics
Possible root piece or bone piece take IOPA x-ray and analgesics

69 Patient getting treatment done under IV Sedation - suddenly not responding has decreased
blood pressure and decreased breathing
Oversedation
Asthma

70 Patient is having epileptic seizures continuously for 10 minutes in dental surgery - first line of
management is
Place in recovery position
Give diazepam
Oxygen
Intranasal or buccal midolazam

pink page 546


71 Sudden unilateral onset of facial swelling and pain over parotid area, which is not yet
fluctuant:
Bacterial saliadenitis
Salivary duct infection
Saliorrhea
Viral siliadenitis

72 What do you see in irritable bowel syndrome?


Mucosal blistering
Herpitiform ulcers
Mucosal tags
Macular patches on buccal mucosa

Irritable bowl syndrome =mucosal ulcer


Irritable bowl disease(crhon)=mucosal tags

73 A fifteen year old patient asks for bleaching of teeth. What is the first line of management?
Discuss options with both patient and parent
Discuss option with both patient and parent, once you have obtained consent from patient
Discuss only with patient
Discuss only with parent
-
74 A child avulsed permanent tooth came with her aunt, what will you do?
Do not do anything
Do not do treatment as aunt is not legal guardian of the child
Do not do treatment as child's mother is not there to give consent
Re-implant and splint it with ​aunt's consent (I have gone for this option, because treatment can
be some times done, if it is in
child's best interest)

75 Best ways to reduce radiation for patient who is going to have IOPA X-rays
Lead apron
Paralleli
Rectangular collimation
D-Film
ng technique

emq
76 Match the option
Apical periodontitis
Lateral apical abscess
Crack tooth syndrome
Dentine hypersensitivity
Hyperemic pulp
Reversible pulpitis
Options
Intermittent pain from molar which is having huge restoration leaking>Reversible pulpitis

Prolonged intense pain for several days


Acute pain which decreases 10-15 minutes after the stimulus is removed
Intermittent pain on biting >Crack tooth syndrome
Intermittent pain with hot things >Hyperemic pulp

77 Match the options


Simple cantilever conventional
Simple cantilever minimal
Spring cantilever conventional
Spring cantilever minimal
Fixed-fixed conventional
Fixed-fixed minimal
Fixed movable
Options to match
Missing upper right central 11 and restored 21>​Simple cantilever conventional

Lateral incisor has restorations, pointed canines>​Simple cantilever conventional

46 and 44 present 45 missing, 44 minimally tilted, 46 heavily filled both are tipped 46mesial and
44 distal>​Fixed movable

78 What is the best cement used to cement minimally done inlay restoration (MOD)
GIC
ZNO
Zinc phosphate
Zinc poly carboxylate
Resin cement
pink p 238

79 What sort of matrix is best for restoring disto-occlusal restoration of 7?


Sectional matrix
Auto matrix

80 Patient had trauma with minimal mobility of front teeth, no symptoms or pain or anything,
what is the best approach
Splinting one week
Splinting two weeks
Splinting 4 weeks
Soft diet and review ​~

81Patient had trauma 8 days ago, upper central incisor palatal luxation mild, not interfering with
occlusion but tooth non vital ,What do you do?
Pulp extirpation +CaoH2 dressing
Reposition
Reposition and splinting

82 Patient had trauma on 14 (vital pulp), only thin buccal cusp remaining, best treatment option
MOD-Onlay
Inlay
Composite

83-Deciduous tooth had intrusion 61, what is most likely to occur for permanent tooth
Impaction
Uneruption
Hypoplasia
Dilaceration
PINK P 73

The traumatically induced type is caused by intrusion of the 1° incisor, resulting in displacement of the
developing 2° incisor tooth germ. The effects depend upon the developmental stage at the time of injury.

84-Splinting time for Avulsion


2 weeks
1 week
7-10 days
pink p108

85 Got a call from mother of a 3-year old child, who had trauma and avulsed the front tooth.
What instruction you will give to mother
Place tooth in milk and attend surgery as soon as possible
Place in the salt water and attend surgery as soon as possible
Place the tooth in its position back and attend surgery as soon as possible
Attend surgery as soon as possible with no special consideration to the tooh

86 A 3-year-old boy avulsed 61, and mother re-implanted it in its socket and clot is holding the
primary tooth. IOPA shows
permanent tooth present, what would you do
Leave it and review:
Splinting for one week
Remove and space maintainence

87 A child has grossly broken down upper and lower molars, carious tooth, which x-ray would
you consider
IOPA
Bitewings
Vertical Bitewings
opg

88 When patient complaints about a treatment and unhappy about the procedure and gives a
complaint, in how many days you have to acknowledge the complaint
1 day
3 days
10 days
20 days

89 Patient has cardiac arrhythmias on warfarin and yesterday had INR of 3, today needs
extraction. What do you do?
Differ the extractions
Alter the dose of warfarin
Do the extraction and control post operative bleeding

90 Match the questions to the options


l)General anesthetics and day surgery
2)General anesthetics and one day inpatient
3)Local anesthetics and oral sedation
4)Local anesthetics and inhalation sedation
5)Local anesthetics
OPTIONS
a)Child 3-years-old, removal of all deciduous primary molars​>1
b)12- years-old very anxious patient, removal of all 5's:​>4
c)Extraction of fully erupted molar in 4-month-old pregnant lady​>5
d)Child for extractions who is medically compromised ​>2

91 Secondary impressions for edentulous patient close fit tray with hyper gap reflex. Which
material you use for taking an impression
Alginate
Plaster of Paris
Zinc oxide eugenol

92 Palatally impacted upper maxillary canines. How would you know? Whether they are
impacted palatally or buccally and which of the following x-rays are best option _
Take 2 Paralleling technique radiographs
Bisecting technique
Bitewings
OPG

parallex tech

94 Which of the following antifungal medication interact with warfarin and enhances its effect?
Ketacanazole
Micanazole
Amphotrocin B

95 A patient came after two days of extraction, complains he had pain for two days, examination
showed localized swelling, no lymphadenopathy. What is the first line of treatment?
Irrigation - curettage of socket, antibiotics and analgesics
Irrigation and antibiotics
Possible root piece or bone piece take IOPA x-ray and analgesics
Irrigation and debridement

97 Patient is having epileptic seizures continuously for 10 minutes in dental surgery - first line of
management is
Place in recovery position
Midazolam
Give diazepam
Oxygen
98 Sudden unilateral onset of facial swelling and pain over parotid area, which is not yet
fluctuant
Salivary duct infection
Sialadenitis
Saliorrhea

99 What do you see in irritable bowel syndrome?


Mucosal blistering​ >​herpitiform ulcers
Mucosal tags
Macular patches

100 A fifteen year old patient asks for bleaching of teeth. What is the first line of management?
Discuss options with both patient and parent
Discuss only with patient
Discuss only with parent

101 A child avulsed permanent tooth came with her aunt, what will you do?
Reimplant ​with aunts consent
Do not do anything
Do not do treatment as aunt is not legal guardian of the child
Do not do treatment as child's mother is not there to give consent

102 Best ways to reduce radiation for patient who is going to have IOPA X-rays
Lead apron
Paralleling technique
D-Film
Rectangulr collimation

104 Match the options


a.tetracycline stain>​Bleaching(chair side}
unless sever then microabrasion
b.mild flourosis >

c.hypo plastic pit


d.Food staining extrinsic stain
Treatment options
Crown
Microabrasion followed by veneer
Bleaching(chair side}
Whiting tooth paste
106 Which test is the most reliable test to indicate the presence of active hepatitis
H bs(surface }antigen
Hbc(core)antigen
Hbe antigen
Antibody to HBs antigen

scully p283
the presence of HBeAg means ongoing viral activity and the ability to infect others, whereas the presence of
anti-HBe signifies a more inactive state of the virus and less risk of transmission.

107 First line treatment to needle stick injury


Wash the area under running water
Serape the area
Refer to infectious disease specialist
pink p 355

108 what is legal amount (unit}of alcohol intake for men


18
21
24
14 delivering better oral health
Department of Health guidelines

● men should not regularly consume more than two to three units per day
● women should not regularly consume more than two to three units per day
● alcohol should be avoided for 48 hours following a heavy drinking session to
allow the body to recover.

109 Medical condition of pt who reserved a seat in a dinner meating for his dead wife
Mania
Anxiety
Depression
Schizphrenia

110 Lady in 40s got TMJ pain for about few months, she got divorced recently what is the first
line treatment
a.amitrptyline
b.​physiotherapy
c.occlusal splint
pink p 460

111 which epithelial cell that responsible for the attachments?

9-what type of cells that proliferate in later stager of her life?


a.Odontoblast
b.cemtoblast
c. rest of malazess
e.undifferentiated mesynchymal cells

112 BLS no of compressions


40:1
30:1
30:2

113 Patient fit and healthy then he got cold and clammy then he lost his consciousness
Syncope
Epilepsy
Hypoglycaemia

114 Drug of choice to acute asthma


Salbutamol
Steroids

115 The term abrasion best describes:


A.Loss of substance by chemical agent (erosion)
B.Loss of substance by external agent (abrasion)
C.Loss of substance by the movement of tooth against tooth (attrition)
D.The rapid loss of substance that is seen in the movement of porcelain crowns aganist natural
teeth (attrition)

116 Some days after preparation and filling of a shallow class I amalgam cavity the patient
complains of pain on biting. You would:
A.Perform vitality test
B.Replace filling
C.Check for premature contacts
D.Remove all occlusal contacts from this filling
E.Tell the patient to wait 2-4 weeks, the pain will go away

117 In a composite filling, the matrix band is for


A.Help shaping and contouring the filling
B.Prevent material to be pushed under the gingival margin

120 In a class 11.2 malocclusion, which bridge design would be contraindicated for a missing
lateral upper incisor?
A.Cantilever bridge
B.Maryland bridge

121 Which is the best cantilever bridge design for missing maxillary canine? Abutment on
A.Both premolars
B.Lateral and central incisor
C.Lateral incisor
D.First premolar

122 A 9 years-old child who has sustained a fracture of a maxillary permanent central incisor in
which 2 mm of the pulp is exposed, presents for treatment 30 minutes after injury. Which of the
following should be considered?
A.Remove 1-2 mm of the pulp tissue surface, place calcium hydroxide and fill with resin
B.Remove 1-2 mm of the pulp tissue surface and cover with ledermix
C.Place calcium hydroxide directly on the exposed pulp
D.Pulpotomy using formocresol
E.Pulpectomy and immediate root filling
pink page100

123 In a flouridated toothpaste with 0.304% sodium fluoride the amount of flouride ions is
A.400 ppm
B.l000 ppm
C.1500 ppm
D.4000 ppm
0.05% NaF=0.023%F=0.23mg F/ml
0.304%NaF=0.14%F=1.4mg F/ml=1400ppm

124 The most important diatary habit for caries development is


A.Amount of sugar intake
B.Frequency of sugar intake
C.Form of sugar intake

125 The normal unstimulated salivary flow rate is


A.0.02 ml/min
B.0.2 ml/min
C. 2 ml/min
Stimulated- 1-2 ml/min
Unstimulated- 0.2- 0.5
Xerostomia- less than 0.1

126 Titanium is used in dentistry


A. ln a very pure form in implants
B. ln an alloy with aluminium in casting for crowns and bridges
C. ln an alloy with nickel in orthodontic wires
D. A and B
E. A, B and C

127 What is the reason for a tooth to develop pulpitis several years after setting of a full veneer
gold crown?
A. Bacterial microleakage

128 How is the regeneration process after damage by injury to odontoblasts working?
A.Proliferation of the remaining odontoblasts
B.Differentiation from fibroblasts
C.Regeneration from undifferentiated mesenchyme I cells
D.Histodifferentiation from ectodermal cells
E.Differentiation from the inflammation cells

129 In construction of full dentures, what does the term "too low vertical dimension" refer to?
A.A situation in which there is too much interocclusal space between upper and lower artificial
teeth when the mandible is in rest position.

130 Reversible pulpitis is characterized by


A.Pain lasts longer on hot or cold stimulus than normally
B.Patient can't localize pain
C.Wil have periapical involvement in radiograph

131 Irreversible pulpitis is characterized by


A.There is often a history of spontanous pain
B.Sudden throbbing pain
C.Pain can't be localised when it reaches the periapical area
D.There is pain which lingers for a short duration after romoval of stimulus
pink p222

132 What are the symptoms of internal resorption?


A.very painful
B.Symptom-free or only mild pain

133 What kind of root fracture in a tooth has the best prognosis? A fracture at the
A.Apical third
B.Coronal third
C.Middle third
D.Vertical fracture
least is vertical then coronal as there's communication with the oral cavity then middle third

135 which of these disease that caused by positive lacto bacillus


Syphilis>​Treponema pallidum
TB> ​Mycobacterium tuberculosis
Angular cheilitis >​ ​combined staphylococcal, β-haemolytic streptococci and candidal infection

Lactobacillus species are also acidophilic and have been implicated in fissure caries

Lactobacillus Secondary colonizer in caries. Very acidogenic. Often found in dentine caries.

136 Child with sore throat and feeling unwell,he got macular rash on his cheek?
a.measles
b.chicken pox
c.herpes simplex
d.​ Infectious mononucleosis (glandular fever)

137 Diabetic pt with abscess complain of failure of root canal therapy and during the
examination the filling was leaking? What the reason
a.lack of coronal seal
b.lack of apical seal
c. Pt medical condition

138 what is the most common site that affected by burning mouth syndrome?
Tongue
Palate
Cheek

139 Pt worried of getting cancer what the advice that should u give?
Regular check up
5 fruit/veg a day

140 Pt got recently metallic taste sensation after taking oral medication?
a.burning mouth syndrome
b.oral dysthesia
c.oral dysgeusia

141 Reasons for crowding


A early loss of deciduous teeth
b.delayed eruption of lower first molar
c primary failure of eruption

143 Reason for delayed eruption of lower first molar


a early loss of deciduous teeth
b.delayed eruption of lower first molar
c primary failure of eruption

142 Reason for palatal position of upper canine


a early loss of deciduous teeth
b.delayed eruption of lower first molar
c primary failure of eruption
d abnormal position of crypt

143 A patient in your dental chair shows chest pain, weak pulse and dysponea, what is your
initial management,
● Administer nitro-glycerine and keep the patient up seated
● Put the patient in supine position
● Wait until the symptoms go away
144 Developer was contaminated with other chemical and was not mixed properly. What is the
effect on the X-ray film?
● Too dark film
● Light film
● Foggy
pink p 748

145 Branchial Cleft cyst is located


● In front of the neck
● On anterior border of the Sternocleidomastoid muscle
● Shows when swallowing
pink p 497
also> ​A ​branchial cleft cyst​ is a congenital epithelial cyst that arises on the lateral part of the neck due to failure of
obliteration of the second branchial cleft (or failure of fusion of the second and third branchial arches) in embryonic
development.

146 What is the approximate unstimulated salivary flow rate,


● 2 ml/min
● 0.2 ml/min
● c. 0.02 ml/min
● D. 20 ml/min

147 Some hours after the extraction of a lower molar the patient complains of prolonged post
operation bleeding and pain, how
would manage this,
● Prescribe analgesics and ask the patient to follow a strict oral hygiene
● Administer 5% Marcaine Local Anaesthetic, prescribe analgesics and pack the socket
with alvogyl
● Administer 5% Marcaine Local Anaesthetic, suture the socket and prescribe analgesics
● Suture and give pressure packs

148 A Gracey curette(​area specific)​ is characterized by


● The blade and the shank form a 90​2​angle
● Can be used on both sides
● Can be used on any tooth surface
● It is specific for each surface ofthe tooth

149 A patient with no positive history came along for scaling. The moment you pick up your
anterior scaler you punch your finger,
what should you do?
● Complete the procedure as if nothing has happened
● Check dentist's blood for Hepatitis B antibody HBsAb
● Check dentist's blood for Hepatitis B antigen HBsAg
● Check patient's blood for Hepatitis B antibody HBsAb and HIV antigen HIVAg
● Check patient's blood for Hepatitis B antigen HBsAg and HIV antibody HIVAb
● Dentist should go and take a HBsAb vaccine

he didnot start yet

150 A patient on the dental chair has cardiac arrest. What is INCORRECT,
● Observing the vital signs and check that the air way is clear is at high importance
● Expired air has 15% 02 only, and cardiac compressions achieve 30-40% of cardiac
output
● Intermittent positive pressure at the rate of 40/min will reduce the chances of cerebral
hypoxia
● Intermittent positive pressure is better than mouth to mouth when it has been given at
the same rate.
● You check the pulse and respiration of the patient before starting any resuscitation

it should b 100-120

151 Treatment for small carious cavity in co operative child


Pulp capping
Pulpotomy
Pulpectomy

Non of the above! Jus simple restoration

152 Treatment for pinpoint exposure in symptomless tooth


Pulp capping
Pulpotomy
Pulpectomy

154 Pt with labially impacted upper canine, how does u know clinically?
Swelling on the labial area
Swelling on the palatal side

155 First line management of dry socket


a.irrigation.x ray.metrondiazole 200
b.irrigation xray metrodiazole 400
c.irrigation and x ray only

156 First line management of pt with palatal dietary erosion


a.crown.,
b.palatal veener
c.direct composite

157 Preferred bridges for missing UR2


A Minmal prepartion adhesive bridge
B Conventional adhesive bridge
C Fixed-fixed bridge
D Minimal preparation fixed fixed
E Simple cantilever
F Spring cantilever

158 Preferred bridges for missing ULl


A Minmal prepartion adhesive bridge
B Conventional adhesive bridge
C Fixed-fixed bridge
D Minimal preparation fixed fixed
E Simple cantilever
F Spring cantilever

158 Preferred bridges for missing UR3


A Minmal prepartion adhesive bridge
B Conventional bridge
C Fixed-fixed bridge
D Minimal preparation fixed fixed
E Simple cantilever
F Spring cantilever

159 Preferred bridge for missing 1st molar


A Minmal prepartion adhesive bridge
B Conventional adhesive bridge
C Fixed-fixed bridge
D Minimal preparation fixed fixed
E Simple cantilever
F Spring cantilever

160 Preferred bridge for missing 1​st​premolar


A Minmal prepartion adhesive bridge
B Conventional adhesive bridge
C Fixed-fixed bridge
D Minimal preparation fixed fixed
E Simple cantilever
F Spring cantilever
161 Preferred local anesthesia option for a normal fit pt?
A Lidocaine + adrenaline
B Prilocaine+felypressine
C Bupivcaine
D Benzocaine
E Articaine

pink p 606
Commonest solution used is lidocaine 2% with adrenaline 1:80 000. A latex-free version is available for
patients with latex allergy.1

162 LA with prolonged duration of action?


A Lidocaine + adrenaline
B Prilocaine+felypressine
C Bupivcaine
D Benzocaine
E Articaine
pink p 606

163 Preferred option of la for pt with heart disease?


A Lidocaine + adrenaline
B Prilocaine+felypressine
C Bupivcaine
D Benzocaine
E Articaine
In patients with severe hypertension or unstable cardiac rhythm, mepivacaine without adrenaline may be
used. Alternatively, prilocaine with or without felypressin can be used but there is no evidence that it is any
safer. Felypressin can cause coronary vasoconstriction when used at high doses; limit dose in patients with
coronary artery disease'

164 Flavoured topical anesthesia?


A Lidocaine + adrenaline
B Prilocaine+felypressine
C Bupivcaine
D Benzocaine
E Articaine

165 which of these jobs dentist can do it?


A radiotion protection supervisor+phsices
b.radiation protection supervisor+practioner
c. practio ne r +refe re r +0 pe rato r

166 Pt on warfarin, what is the accepted inr to do simple extraction


A2
B3
C4
D5

167 which of this pt has risk of dry socket


a.smoker
b.pt on warfarin
c.elderly pt

168 what is the most reasonable indication for third molar removal
A First episode of pericorinitis
B Second episode
C Periapical infection with erupted wisdom tooth

169 what the feature of the carpet of surgery?


no carpet.impervious floor

170 Dentist did root canal treatment previously to child pt and now he do a crown for the same
pt what band the dentist should claim?
Band 1
Band 2
Band 3
No calim

171 what ur advice to childs mum whos her child s teeth knocked out?
a.Reimplant it
b.keep it in the milk and come as soon as possible correct
c.keep it in the water
if permenent tooth

172 Child accidentally consumed 5mg/kg flouride what the first line treatment?
Give child salty drink
Give child sugary drink

173 What is the most correct flap design in apecictomy?


Apical re position
Apex is broader than the base
Base is broader than the apex
Apex and base are equal

174 Common irrigation solution for endodontices?


A sodium hypochlorite
B Saline
C Chlorhexidine

175 What will develop after prophylaxis?


A.Aceliular pellicle is formed immediately after
B.Celiular pellicle is formed immediately after
C.Acellelar pellicle is formed after 48 hours
D.Cellular pellicle is formed after 48 hours

176 What is the purpose of making a record of protrusive relation and what function does it
serve after it is made?
A.To register the condylar path and to adjust the inclination of the incisal guidance.
B.To aid in determining the freeway space and to adjust the inclination of the incisal guidance.
C.To register the condylar path and to adjust the condylar guides of the articulator so that they
are equivalent to the condylar paths of the patient.
D. To aid in establishing the occlusal vertical dimension and to adjust the condylar guides of the
articulator so that they are equivalent to the condylar paths of the patient.

177 A 50 years-old patient presents with pain from time to time on light cervical abrasions. What
is your first management to help patient in preventing pain in the future?
A.Change diatary habits
B.Change brushing habits
C.GIC fillings

178 In planning and construction of a cast metal partial denture the study cast
● facilitates the construction of custom trays
● minimizes the need for articulating
● provides only limited information about inter ridge distance, which is best assessed
clinically
● can be used as a working cast when duplicating facilities are not available

179 What are the artificial teeth in removable dentures made of?
A.Porcelain
B.Cross-linked methyl-methacrylate
C.Ethyl-methacrylate
D.Acrylic
Confused between b and d
As acrylic is ( poly methyl methacrylate ) and it should be cross linked
180 Following extraction of the molar teeth
● The ridge height is lost more from the maxilla than from the mandible
● The maxillary ridge will get more bone lost from the palatal aspect than the buccal
● The mandibular arch is relatively narrower than the maxillary arch
● Compared with the pre-resorption state, the mandibular ridge will lose more bone from
the lingual aspect than the buccal one.

181 Which anatomical landmark is important to include in impressions for lower full dentures?
A.Mylohyoid ridge
B.Lower incisive papilla

182 Wrought metal is to be,


● Marble
● Quenched > casting alloy
● Has undergone cold treatment during processing

183 Which ofthe following is ONE indication for indirect pulp capping?
A.When further excavation would lead to pulp exposure
B.Excavation of a very deep caries
184 How does fluoridation work in theory?
A. Fluoride ions are integrated by changing Hydoxylapatite to Fluorapatite

185 A major difference between light cured and chemical cured composite is that during setting
or in function the light cured materials tend to:
● Seal the margins better and completely
● Exhibit less wear on time
● Undergo greater colour change
● Shrink more rapidly
● Posses greater fracture toughness

186 What consideration is important in deciding if a bridge for upper missing incisors should be
made in pontic design or with gingiva imitation?
A.Wishes of patient
B.Bone resorption in edentoulos span

187 The most common cause of porosity in porcelain jacket crowns is,'
● Moisture contamination
● Excessive firing temperature
● Failure to anneal the platinum matrix
● Excessive condensation of the porcelain
● Inadequate condensation ofthe porcelain

188 How should the occlusion in partial removable dentures be designed?


A.Artificial teeth should be out of occlusion
B.Artificial teeth should not interfere with the incisal guidance established by the remainig
natural teeth.

189 The minimal labial tooth reduction for satisfactory aesthetics with porcelain fused to metal
crown is,
A.1mm
B.The full thickness of enamel
C.1.5 mm
D.2.5mm
E. One third of the dentine thickness

Preparation requires 0.5mm reduction of lingual surface with chamfered margin and
labial reduction of 1.2–1.5mm, with shoulder.

190 In removable partial denture, the principle of an indirect retainer is to:


● Stabilise against lateral movement
● Prevent settling of major connectors
● Restrict tissue movement at the distal extension base of the partial denture
● Minimise movement of the base away from the supporting tissue

191 When a removable partial denture is terminally seated the retentive clasps tips should:
A. Apply retentive force into the body of the teeth
b.Exert no force
c.Be invisible
d.Resist torque through the long axis of the teeth

192 Glass lonomer Cement sets because of,


● Acid-Base reaction
● Addition polymerisation reaction
● Growth of glass crystals
● Slip plane locking
● Solvent evaporation
CHIrchil p 124
Setting reaction
Acid–base reaction between glass and polyalkenoic acid.

193 The reflex in gagging patients is caused by:


● Trigeminal nerve
● Glossopharyngeal
● Facial nerve
● Recurrent laryngeaI

IX Glossopharyngeal.​ Supplies sensation and taste from the posterior 1⁄3 of the
tongue, motor to stylopharyngeus, and secretomotor to the parotid. Lesions
impair the gag reflex in conjunction with X.
​X Vagus. Has a motor input to the palatal, pharyngeal, and laryngeal muscles.
Impaired gag reflex, hoarseness, and deviation of the soft palate to the
unaffected side are seen if damaged. The vagus has a huge parasympathetic
output to the viscera of the thorax and abdomen.

194 The use of nickel chromium in base plate should be judiciously considered because:
● A significant number of females are allergic to nickel .,
● A significant number of females are allergic to chromium
● A significant number of males are allergic to nickel
nickel is a possible carcinogen and common allergen.
​NB Some patients are sensitive to nickel.
195 Which of the following liquids is not suitable for prolonged immersion of cobalt chrome
partial dentures:
A. Alkaline peroxidase
B. Sodium hypochlorite
C.Soap solutions
D.Water
pink p 306

196 In complete dentures, cheek biting is most likely a result of:


● Reduced Overjet of posterior teeth
● To high vertical dimension
● Teeth have large cusp inclines
pink page 308

197 The most common cause of fracture at the isthmus of a class II dental amalgam restoration
is:
A.lnsufficient condesation
B.Fracture line developing from pulpal-axial angle of the cavity
C.Underconturing of the isthmus area
D.Moisture contamination of the amalgam during placement
E.lnadequate bulk of amalgam at pulpo-axialline angle
pink p239

198 Why do you overpack amalgam fillings?


A. To remove excess mercury
B To prevent microleakage
pink p 631
Condensation Carry out incrementally by hand instruments (lathe-cut or spheroidal).
Preparations should be overfilled so that the Hg-rich surface layer is removed by carving.

199 What is CORRECT in regard to the periodontal surface area in maxillary teeth:
● central incisor> first premolar> second premolar
● Canine> first premolar> central incisor
● Canine> lateral incisor> second premolar
● Canine> central incisor> first premolar
pink p 270

200 When restoring with composite resins, why do we do the cavo-surface bevelling:
A.Aesthetic
B.To open enamel rods for acid attack
C.To smooth preparation
D.A and B
E.AII of the above

201 A well constructed complete denture:


● Needs little maintenance
● Needs less than a week for adjustment and total success
● Has adverse effects and decreases taste sensations
New denture even if constucted well is always a greatest cause for max complains from patient in first few weeks.
As new,
they cant adapt to it.
Their automated chewing reflex and crushing food takes 4 to 6 months to fully get used to.
New denture usually causes ulcers
If upper , Covers palate and blocks taste buds.

202 On examination of a composite restoration you find a dark stain:


A. Replace the composite
B.Repair with unfilled resin
C.Apply topical fluoride at the margin

203 A patient complains of sensitivity, on examination you find a composite filling restoring a
good cavity preparation without any secondary caries; what is your next step:
● Extirpate the pulp that is obviously inflamed
● Place ZOE dressing to sedate the pulp
● Ask patient to come back in six months
● Repeat restoration

204 What is the best way to cement a Maryland bridge,


A.GIC
B.Resin cement
C.Composite resin
D.Zinc Phosphate cement
E.Oxide Zinc and eugenol

205 The ideal length of a post in the fabrication of crown and core of endodontica!ly treated
tooth is:
A.2/3 of tooth
B.the tooth length
C.1.5 times that of the crown
D.Yz root length
E.The length of the crown
PINK P 256
As a general guide the post should be at least equal to the anticipated crown height, but
a minimum of 5mm of well-condensed GP should be left. A periodontal probe or silicone stop is
helpful to check prepared canal length

206 While you finish a class I cavity, the enamel is sound but you notice a thin brown line ln the
dentine and on the dentino-enamel junction, what is your response,
● You leave it and complete the final restoration
● You extend your preparation and clean it
● You apply a cover of varnish

207 Dental caries ofthe proximal surfaces usually starts at,


● Somewhere between the ridge and the contact area
● Just gingival to contact areas
● Just above the gingival margin
● At the contact point

208 The Frankfort plane is defined by which anatomical landmarks,


● Porion, orbitale
● Sella, orbitale
● Nasion, Tragus
Frankfort plane Line joining porion (superior aspect of external auditory
meatus) with orbitale (lowermost point of bony orbit).

209 The biting load of a denture base to the gingival tissues compared to teeth are,
● Ten times more
● Ten times less
● Equal

210 The difference between normal stone and the die stone is,
● In the size and shape of the particles
● The mixing

211 The advantage of the silicone in soft relining material over hard plastic acrylic materials is,
● Capability to flow
● Prevents the colonization of Candida albicans
● Resilient in long run
● Better bond strength
PINK P 662

212 A female patient comes to you complaining of persistent pain in a heavily restored central
incisor; you suspect irreversible pulpitis and you have been told that she is in transit leaving by
plane next day. Your treatment will be,
● Remove filling and place a sedative dressing
● Pulpectomy and Ledermix dressing
● Pulpectomy and calcium hydroxide dressing
● Prescribe analgesics and systemic antibiotic
pink 223

213 The flexibility of the retentive clasp arm does not depend on:
● Length of the arm
● The cross section shape
● The material used
● Degree of taper
● The exerted force

214 Following calcium hydroxide pulpotomy, the dentist would expect dentine bridge to form at,
● The exact level of amputation
● Level somewhere below the amputation
● Half way between amputation and apex
● At the apical region of the tooth
Calcium hydroxide is generally accepted as the material of choice for pulp capping. Histologically there is a complete
dentinal bridging with healthy radicular pulp under calcium hydroxide dressings.When calcium hydroxide is applied
directly to pulp tissue there is necrosis of adjacent pulp tissue and an inflammation of contiguous tissue. Dentinal
bridge formation occurs at the junction of necrotic tissue and vital inflamed tissue. Beneath the region of necrosis,
cells of underlying pulp tissue differentiate into odontoblasts and elaborate dentin matrix

215 In the construction of a full veneer gold crown, future recession of gingival tissue can be
prevented or at least minimised by,
● Extension of the crown 1 mm under the gingival crevice
● Reproduction of normal tooth incline in the gingival one third of the crown
● Slight over contouring of the tooth in the gingival one fifth of the crown
● Slight under contouring of the tooth in the gingival one fifth of the crown

216 What is correct in regard to high copper amalgam,


● Reacts and strengthens the amalgam by its dispersion properties
● Reacts to form copper-tin phase thereby eliminating the tin-mercury phase
● Reacts to form copper-silver phase thereby eliminating the silver mercury phase
● Reacts and strengthens the amalgam by its grain diffusion.

For copper-enriched alloys the reaction may be represented by:

Ag3Sn + Cu + Hg → Ag2Hg3 + Cu6Sn5 + Ag3Sn or γ+Cu+Hg→γ1 +Cu6Sn5 +γ

The essential difference between this and the reaction for conventional alloys is the
replacement of the tin–mercury, γ2 phase in the reaction product with a copper–tin phase.

217 The removable partial denture requires relining,what would be the most appropriate action,
● take an impression by asking the patient to occlude on it
● Provide equal space between denture and gingival tissues.
● Make sure the framework and retainers are seated in place before taking impression



218 Stiffness of material is measured by
● Proportional unit
● Modulus of elasticity
● Stress/ strain
● Ultimate tensile strength
pink

219 Two central incisors on a radiograph are showing with what looks like eye drop
radiolucency. You decided to start endodontic treatment on these teeth but when you tried to
open access to the root canal you find clearly closed orifices with what look like secondary
dentine. What is your initial management?
● Leave as it is and start a permanent restoration.
● Start systemic antibiotic
● Try to ream and file canals

220 After the initial development stage and in the absence of pathology, the size of the pulp
chamber has been reduced by,
● Deposition of primary dentine
● Deposition of secondary dentine
● Reparative dentine
● Pulp fibrosis
● Deposition of reparative dentine

223 Denture stomatitis is commonly associated with,


● The continuous wearing of removable orthodontic appliances in otherwise healthy
patient
● The proliferation of hypertrophic tissue at the denture periphery
● The overgrowth of some constituents of oral normal microflora
● Allergy to denture base material

224 The light emitted by the polymerization lamp has to be checked from time to time. The
meter used for this only measures light
in the range of:
● 100-199 nm
● 200-299 nm
● 300-399 nm
● 400-499 nm
225 Which is correct in regard to shade selection of crowns:
A. It should be selected before starting preparation
b.Chroma is the lightness/darkness of colours
c.Value is the colour itself
d.Hue is the concentration of colours

226 Where would you expect to find the mylohyoid muscle in relation to the periphery of a full
lower denture:
A.Mandibular buccal in the midline
B.Mandibular lingual in the first premolar area
C.Mandibular lingual in the midline
D.Mandibular disto buccal area

227 After reimplantation of an avulsed tooth the prognosis may be poor because of
A.External resorption
B.lnternal resorption

228 2.2 mg of NaF contains how many mg of fluoride ions?


A.O.5mg
​B.1.0 mg
C.1.5mg
D.10mg
Each ReNaf Fluoride Chewable Tablets 1 mg contain 1 mg fluoride ion (F-) from 2.2 mg
sodium fluoride (NaF).
Each ReNaf Fluoride Chewable Tablets 0.5 mg contains 0.5 mg F- from 1.1 mg NaF.
Each ReNaf ReNaf Fluoride Chewable Tablets 0.25 mg contains 0.25mg F- from 0.55 mg NaF.

229 How would you treat denture stomatitis?


A​.Nystatin
B.Tell the patient to leave the denture out for some days

230 What is true about third molar surgery?


A.Swelling is maximum after 24 - 48 hours
B.Mental paraesthesia indicates careless technique

231 1n anaesthetizing a 70 kg healthy man with Lignocaine 2% with 1:100,000


vasoconstrictor.what is correct?
A.The toxic threshold is 22 ml
B.2.2 ml is the maximum you can give in one session
C.Lignocaine has the same anaesthetizing capacity as Benzocaine
D.Lignocaine is 5 times less potent than Bupivacaine
E.Lignocaine is more dangerous in Hypothyroidism than Bupivacaine

232 A suddenly swollen upper lip that lasts for 48 hours or more is most likely
A.Haemangioma
B.Agioneurotic oedema
C.Mucocele
D.Cyst

233 What is the most important factor to reduce radiation in dental radiographs?
● Speed of film
● Collimation
● Filtration
● Cone shape and length
● Use of lead apron

234 What is the best way for a permanent decline in caries of a population?
A.Change diatary habits
B.Topic and water fluoridation
C.Awareness of dental health matters
D.Better tooth brushing

235 Loss of sensation/paraesthesia in the lower lip may be produced by,


● Bell's palsy
● Traumatic bone cyst
● Trigeminal neuralgia
● Osteomyelitis
● Ludwig's angina

236​ In anxoius and psychologically stressed patients gingivitis is often more severe
because of
A.Stress causes histamine and serotonine release
B.5tress causes catecholamine and corticosteroid release
c.Stressed people neglect their oral hygiene

237 A retained lower primary incisor causes the permanent incisor to


A.erupt buccally
B.erupt lingually
C.ankylose
238 What is the least probable consequence in thumb-sucking?
A.Reclining of lower incisors
B.Protrusion of upper incisors
C.Formation of deep palate with big overbite

239 Ankylosis of teeth is often found after changes in the continuity of the occlusal plane. These
changes are caused by
A.Overeruption of opposing teeth
B.Localised growth inhibition of the alveolar bone
C.Sinking of ankylosed tooth into the bone

240 What is not important in obduration materials for primary teeth?


A.Good apical seal
B.Radioopacity
C.Antibacterial
D.Resorbable

241 What is your first consideration in the treatment of dry socket?


A.Prevention of osteomyelitis
B.Pain relief

242 A patient presents with pain in the upper left segment. On inspection you f~nd a localized
alveolar abcess distal 27. What will be you management? -
A.Drainage
B.Extraction of tooth

243 What does not help in establishing the caries risk in children?
A.History of caries
B.Lactobacilius count
C.Dietary habits
D.Brushing habits
E.Genetic predisposition

244 Opioid drugs are similar to which endogenous substances?


● Bradykinins
● Peptides
● Prostaglandins
● Serotonins
● Enkephalins

245 Which of the following have a tendency to recur if not treated correctly?
● Giant cell granuloma
● Lipoma
● Fibrous epulis
● Haematoma
● Pulp polyps

fibrous epulis
Fibrous nodules should be excised together with the small base of normal tissue from which they arise. In
the case of a fibrous epulis, the underlying bone should be curetted. There should be no recurrence if this is
done thoroughly and the source of irritation is removed.

246 When no radiation shield is available, the operator should stand out of the primary x-ray
beam at a distance from the patient's
head of at LEAST:
A.0.5 metres
B.1 metre
C.1.5 metres
D.2 metres
E. 3 metres

Exposure switches should be positioned so that the operator can remain outside the controlled area
and at least 2m from the X-ray tube and patient

the controlled area>​1.5 m in any direction from the patient and tubehead and anywhere in the line of the
main beam until it is attenuated by a solid wall.

247 Which of the following is a typical consequence of dental crowding, assuming no primary
tooth has been lost prematurely?
● Overlapping of lower incisors
● Palatal displacement of upper canines
● Impaction of 15 and 25 between first premolars and first molars
● Mesial tipping of 16 and 26
● Rotation of 16 and 26

248 What is the dominant microflora in acute necrotic ulcerative gingivitis (ANUG)?
● Spirochaetes and fusobacterium SP
● Spirochaetes and eikenella corrodes
● Polymorphs and lymphocytes
● Actinobacillus actinomycetes comitans oral capnocytophaga
● Porphyromonas gingivalis and prevotella intermedia

249 Which of the following is true regarding gingivosis (Desquamative gingivitis)


● It is caused by hormononal imbalance
● Is seen only at or after menopause
● Is frequently caused by lichen planus
● Is a variant pregnancy gingivitis
● Is related to nutritional disturbance

250 What are the points that determine the facial line in cephalometric points
● Nasion, pronasale, pogonion.
● Sella, nasion, pogonion

251 A 10 year old boy presents with small greyish white lesion surrounded by a red halos on the
soft palate and tonsillar pillars, small vesicles are found. He has fever and pain in the ear and
won't eat. The MOST probable diagnosis is?
A.Herpangina
B.Measles
C.Primary herpetic stomatitis

Herpangina Febrile illness with sore throat due to ulcers on soft palate and throat.
Usually lasts about 3–5 days. Rx: soft diet.

252 A 12 years-old child presents with symptoms of widespread gingivitis with !lleeding and
general malaise for several weeks. How would you manage this patient?
● Prescribe Metronidazole 100mg
● Locally debride, give oral hygiene instruction and prescribe H202 mouth wash.
● Give a prophylaxis with ultra sonic scaling
● Refer for haematological screening >​leukemia
● Advise for bed rest with supportive and palliative treatment

253 What is the effect of office dental prophylaxis of regular six month intervals on children's
oral health?
● Reduced caries incidence by approximately 30%
● Provide a long term improvement in oral hygiene
● Provide a short term improvement in oral hygiene
● Prevent gingivitis
● Reduce the need for patient cooperation

Professional intervention improves oral hygiene on short term basis. And dboh too says most effective and long term
improvement provided by tooth brushing twice daily and patient adopting better oral hygiene.

http://www.incredibleimplants.com/procedures/periodontics/prophylaxis-teeth-cleaning
254 What is the most frequent cause of pain which occurs several days after obturation?
● Entrapped Bacteria in the periapical region
● Underfilling the root canal system
● Overfilled root canal

The following factors need to be considered should pain occur following sealing of the root canal system.

High restoration
Overfilling
Underfilling
Root fracture
Once obturation of the root canal space has been completed, restoration of the rest of the tooth can be carried out.
The occlusion must be checked for interferences, to avoid an apical periodontitis, or worse, a fractured tooth.

Root fillings that are apparently overfilled do not as a rule cause more than mild discomfort after completion. The
most likely cause of pain following obturation of the root canal space is the presence of infected material in the
periapical region. The significance of an underfilled root canal is whether the canal has been properly cleaned and
prepared in the first instance, and infected debris is still present in the canal. Post-endodontic pain in these
circumstances may well be due to inadequate debridement of the canal.

Removal of an overextended root filling is rarely completely successful and the options left are as follows:

Prescription of analgesics and, if the pain is more severe and infection is present, antibiotics.
An attempt at removal of the root filling and repreparation of the root canal.
Periradicular surgery and apicectomy

255 A diabetic patient with moist skin, moist mouth and weak pulse; what would you do:
● Give glucose orally
● Administer 02
● Administer adrenaline
● Inject insulin

256 A patient has developed a sever chest pain and difficulties in breathing while in the dental
chair. Your initial response is:
● Administer glycerine trinitrate and monitor patient in upright position
● Patient has an acute episode of angina as demonstrated by curve in ECG
● No treatment is required until confirmed as MI by ECG
● Patient has myocardial infarction as confirmed by ECG

257 In the case of malignant melanoma occurring intraorally, which of the following is true:
● Uncommon on the palate
● Should not be biopsied, as this will increase metasis
● The 5 years survival rate is 20%
● The incidence of oral melanoma is the same as those on the skin
E. Commonly occurs intra orally
cowson

258 What is NOT TRUE in relation to the use of diazepam for sedation?
● Patient commonly complain of post operative headache >F
● An acceptable level of anxiolytic action is obtained when the drug is given one hour
preoperatively >T
● There is a profound amnesic action and no side effects >T
● Active metabolites can give a level of sedation up to 8 hours post operatively >T
● Can be used safely for children >T but might hyperactive or ineffective

Benzodiazepine: gives sedation with amnesia but no analgesia. Give slowly intravenously in
2.5-mg increments until ptosis begins (i.e. eyelids begin to droop; Verrill’s sign) – rapid injection
may cause respiratory depression. Then give local analgesia.
Disadvantages:
May cause pain or thrombophlebitis
Drowsiness returns transiently 4–6h postoperatively due to metabolites desmethyl diazepam,
triazolam and oxazepam, and enterohepatic recirculation
May cause mild hypotension and respiratory depression.
If pre-medication has to be given, benzodiazepines are useful because of their anxiolytic and amnesic
actions, their relative freedom from side-effects and their wide safety margin. Diazepam, lorazepam and
temazepam are used more and more widely for pre-medication. scully

Diazepam has a long half-life (0.8–2.25 days) and its active metabolites have long half-lives (i.e.
N-desmethyldiazepam [1.6–4.2 days]; nordiazepam [about 8 days]).

Diazepam is metabolized to products that have sedative actions (desmethyldiazepam, temazepam and
oxazepam) and thus effects can persist, with a half-life of 20–72 hours.

259 Patient on anti-coagulant therapy requires an extraction to be performed. Which of


the following is NOT true:
● Minor bleeding can be reduced somehow by using tranexamic acid
● Prothrombin value above 2.5 is required to perform extraction
● It takes up to 12 hours for Vitamin K reverse effects of warfarin
● Heparin can be administered sub-cutaneous and acts rapidly

vitamin K can be expected to reverse warfarin 4 – 6 hours after administration.


260 A physician refers a nine year old boy to you to confirm diagnosis. The boy has a fever of
40°C and coughing. When you focus your light into his eyes he turns away. Intra-orally there are
white spots surrounded by red margins. The disease and lesions are:
● Measles and the spots are Koplik's spots
● AHGS vesicles
● Rubella and the spots are Fordyce's spots
http://www.nhs.uk/conditions/measles/Pages/Introduction.aspx

261 What is true in TMJ dysfunction therapy?


A.Should be treated surgically
B.Appliances that raise the bite usually relief the symptoms and are used prior to any surgery

262 What is true regarding pregnancy gingivitis?


A.lt is due to increased gingival microcirculation
B.Elevated oestrogen and gestagen levels are directly responsible .;
C.Hormonal changes cause the growth of anaerobic bacteria (Prevotella interrnedia)

Normal female hormones, of which progesterone is one, are elevated during pregnancy and also in women who are
taking certain birth control pills containing high levels of progesterone. In the presence of periodontal disease,
progesterone stimulates the production of substances called prostaglandins, which cause inflammation of the blood
vessels in the gum tissues.
Warning signs of periodontal (gum) disease include swelling, redness, bleeding and sensitivity of the gum tissues,
common in pregnancy and known as pregnancy gingivitis (gingival – gum tissue, itis – inflammation of). The tiny
blood vessels of the gum tissues become dilated (widened) in response to the elevated hormone levels of pregnancy
and therefore more susceptible to the effects of plaque bacteria and their toxins. Gingivitis is especially common
during the second to eighth months of pregnancy.
Occasionally, overgrowths of gum tissue called “pregnancy tumors” appear on the gums during the second trimester.
Tumor means nothing more than swelling or growth; pregnancy tumors are benign and not cancerous. They are
usually found between the teeth and are believed to be related to excess dental bacterial plaque. They bleed easily
and are characterized by a red, raw-looking mulberry-like surface. They are often surgically removed after the baby is
born if they do not resolve themselve

263 5 mm probing depth means:


A.Patient has periodontitis
B.Probe is 5 mm beyond gingival margin
C.Probe is 5 mm beyond dentino-enamel junction

264 A middle aged woman gives a history of intermittent unilateral pain in the submandibular
region, most probable cause is,
● Calculus in the salivary duct resulting in sialolithiasis. .
● Ranula >typically painless
● Cyst
● Mucocele >mainly affect the minor salivary glands
265 By which of the following mechanism reduces Aspirin pain:
● It is anti inflammatory by the release of histamine
● It blocks the cyclo-oxygenase pathway.

266 In minor oral surgery, what is TRUE in regard to antibiotics:


A.Amoxil satisfactorily covers the dental spectrum
B.Metronidazole and Amoxil have the same penetrating power
C.It is evident that it will reduce post operative swelling
D.There is convincing evidence that Prophylactic prescription of antibiotics will reduce
postoperative infections
E.Most oral infections get anaerobic after 2 to 3 days

267 A patient comes with a firm, painless swelling of lower lobe of parotid which has grown
progressively for the past year. He complains of paresthesia for the past 2 weeks. This is most
likely to be:
● Pleomorphic adenoma
● Carcinoma of the parotid
● Lymphoma of parotid

268 What is true in treating a patient with secondary herpes simplex:


● Acyclovir inhibits viral transcription when applied in the prodromal phase
● Idoxuridine is better than acyclovir when applied topically
● Antivirals are contraindicated in immuno-compromised patient
269 During extraction of a maxillary third molar the tuberosity is fractured; however, it remains in
place attached to the mucoperiosteum. Which of the following procedures should be employed:
● Remove the tuberosity and suture
● Leave the tuberosity and stabilize if required
● Remove the tuberosity and fill the defect with Gelfoam then suture.
● If fractured tuberosity is greater than 2 ern, leave in place and suture

270 An incision biopsy of an ulcerated and indurated clinically suspicious lesion in a 50year old
female reveals chronic inflammation;
you would:
● Inform the patient and her physician of your findings and instruct the patient to return in
six months
● Surgically excise the entire lesion since you know it is not malignant
● Dismiss the patient with instructions for warm saline rinses and re-examination
● Repeat the biopsy
U repeat biopsy seen suspicious.
In incisional biopsy u take a part of lesion as specimen.
So u might hav missed da malignant part while sampling

271 Reducing the size of the focal spot will:


● increase sharpness
● increase density

272 The initial priority in treatment of horizontal fractures is:


● Preservation of pulp
● Immobilisation
● Root canal treatment
● Calcium hydroxide treatment

273 Which of the following has proven to be the MOST important in community preventive
program:
● Dental awareness of the community
● Institution of oral hygiene measures
● Water fluoridation

274 What effect has placing a sealant over pits and fissures on the progression of caries?
l​.Decreased new caries
2.lncreased new caries
3.Progression of exististing caries
4 No effect on existing caries

275 In advanced periodontitis with marked mobility, teeth may be splinted:


A.To improve comfort for the patient
B.splinting helps in transmitting the force to the adjucent teeth to reduce the load on the
involved teeth

276 Swallowing will aid in the diagnosis of:


● Branchial cyst
● Thyroglossal duct cyst
● Ranula
● Retention cyst
pink p.496

277 Which of the following will increase sharpness:


● Larger focal spot
● Smaller focal spot
● Increase object-film distance
● Decrease patient-source distance
eric
what increase sharpness :1- increase source object distance 2- decrease object film distance 3-
smaller focal spot

278 In severe periodontitis the probe will eventually be:


● prevented to go deeper by calculus
● beyond connective tissues in the junctional epithelium
● at the end of the junctional epithelium
● Touching the middle of junctional epithelium
● Touching sulculuar epithelium

279 A 58 year old male has been treated with radiation for carcinoma of tongue. The patient
complains of pain associated with poor dentition. The dental management would be:
A.Immediate extraction of any poor teeth under local anaesthetic with antibiotic coverage
B.Segmental dental clearance and closure to eliminate problems
C.No dental treatment may be due to neuronic of neoplasms
D. Clearance of poor dentition followed by hyperbaric oxygen treatment plus a primary closure
of wounds under antibiotic coverage
E. No extraction as radionecrosis is an important sequelae

280 Which of the following is NOT true about anticoagulation therapy?


● INR of 3 is enough to start any extraction
● Affects extrinsic system and increases prothrombin time
● Heparin can be given subcutaneously and acts rapidly
● It takes at least 12 hours for Vitamin K to reverse the effects of coumarin
Hepar​in acts slowly when given subcutaneously
281 in a radiograph the roots of the upper teeth are too short because of:
● Inadequate horizontal angulation
● Too high vertical angulation
● Too small vertical angulation
eric

282 Characteristic of Squamous Cell Carcinoma of the tongue


● more in white skinned people
● more in alcohol drinking smoking males
● associated with Plummer-Wilson-Syndrome

283 Characteristic of Squamous Cell Carcinoma of the lips


● It reacts far simply to radiotherapy
● metastizes mainly by blood
● relatively rare in Australia

Sq cell Ca of lip can spread through tissue (local), the lymph system (lymphatics), and the blood ( disimeniated mets)
Best treatment modality for lip Ca is surgucal excision and reconstuction not radiotherapy.
It is v much common in australia due to incease uv rays.
in pink book it says SCC mainly spreads by lyphatics and invasion and blood only in later stage.

284 Which type of dentin is not formed due to pulp pathology?


A.Reparative dentin
B.Secondary dentin
C.Primary dentin
D.Reaction dentin
E.Tertiary dentin

285 Which is not true in sickle cell anaemia?


A.Deformed cells with less oxygen transport capacity
B.Higher infarction risk
C.Have wide bone marrow spaces with narrow trabeculae in the alveolar bone of oral cavity
D.Resistant to malaria parasites
E.More common in mediterranean people
it's more common in African and least common in Mediterranean ppl
286 Normal prothrombin time and elevated partial thromboblastin time is seen in
A.Factor VIII deficiancy (Haemophilia)
B.Thrombocytic pupura
C.Leukemia
D.Von Willebrand disease
287 Which of the following describes best a 9 years-old child permanent dentition?

612CDE
6123DE

288 What is the best reason to promote tooth brushing to the public?
A.Less fissure caries
B.Less gingivitis
C.Gingival massage
Personal hygiene care consists of proper brushing and flossing daily. The purpose of oral hygiene is to
minimize any etiologic agents of disease in the mouth. The primary focus of brushing and flossing is to
remove and prevent the formation of plaque or dental biofilm. Plaque consists mostly of bacteria.As the
amount of bacterial plaque increases, the tooth is more vulnerable to dental caries when carbohydrates in
the food are left on teeth after every meal or snack. A toothbrush can be used to remove plaque on
accessible surfaces, but not between teeth or inside pits and fissures on chewing surfaces. When used
correctly, dental floss removes plaque from areas that could otherwise develop proximal caries but only if
the depth of sulcus has not been compromised. Other adjunct oral hygiene aids include interdental brushes,
water picks, and mouthwashes.​However oral hygiene is probably more effective at preventing gum
disease (periodontal disease) than tooth decay.​ Food is forced inside pits and fissures under chewing
pressure, leading to carbohydrate-fueled acid demineralisation where the brush, fluoride toothpaste, and
saliva have no access to remove trapped food, neutralise acid, or remineralise demineralised tooth like on
other more accessible tooth surfaces food to be trapped

289 What is untrue about diabetes?


A.Hypoglycaemia is more common than hyperglycaemia> true
B.lnsulin-dependend patients are of more concern than non insulin-dependend> true
C.Adrenalin causes a decrease in the blood glucose level
290 Which of the following is a feature of Streptococcus mutans?
A.lt does not require a special environment to grow
B.lt can be easily transported from one part of the oral cavity to another
C.lt has the ability to restructure carbohydrates

291 What does the term "caries prevalence" mean?


A.The total number of carious areas affected and any present caries
B.The individual risk for a patient to acquire caries

292 Which of the following are features of herpetic gingivostomatitis?


● Irritability
● Fever
● Occurs in teenagers
● Vesicles occur only on buccal mucosa and tongue

293 A patient in your dental chair suddenly becomes agitated with shallow breathing, full pulse
and a blood pressure of 150/80.
You would
A.Give oxygen
B.Give insulin
C.Give glucose
D.Place patient in supine position
294 The principle clinical sign of active bruxism is:
● Head and / or neck pain
● Excessive tooth wear
● Temporomandibular joint clicking
● Sensitive teeth
● Cheek ridging and tongue scalloping ​(most answers )

295 An adult patient attends your practice complaining of pain-and swelling aSsOCiated with a
previously restored upper first premolar tooth. The pain has been present for a number of days
and is no longer responding to analgesics. His dentition is otherwise well maintained and his
periodontal health is good.
What is the most appropriate approach to treatment?
● Antibiotics and analgesics.
● Extract the tooth
● Carry out a pulpotomy and temporary dressing.
● Carry out a pulpectomy. Temporary dressing
● open Drainage

296 A patient reports that his post crown has fallen out. This crown had been present for many
years. You note that there appears to be a hairline vertical fracture of the root. The tooth is
symptomless.
What is the most sensible approach to treatment?
A.Replace the post crown using a resin-reinforced glass ionomer material
B.Replace the post crown using a polycarboxylate cement
C.Replace the post crown using a dentine bonding agent and a resin-reinforced glass ionomer
material
D Replace the post crown using a resin composite luting agent
E. Arrange to extract the tooth

297 A patient says that he does not like the appearance of his previously root filled upper
central incisor tooth. His dentition is otherwise well maintained and his periodontal health is
good. The tooth appears to be darker than the adjacent teeth.
What is the most appropriate approach to treatment?
A.Provision of a post crown
B.Provision of an all ceramic crown
C.Provision of a metal bonded to ceramic crown
D Non vital bleaching
E. Provision of a porcelain veneer

298 A 25 year old male attends for the first time complaining of sensitivity of a number of teeth.
On examination, the occlusal surfaces of all the teeth are worn with obvious wear facets on the
canines and premolars. Posterior amalgam restorations are proud of the surrounding tooth.
What would be the first stage management?
● Take impressions for study models
● Prescribe fluoride mouth rinse
● Replace the amalgam restorations
● Dietary analysis

299 A 35 year old male patient who admits to grinding his teeth at night has a number of
wedge-shaped cervical (Class V) lesions on
his upper premolar teeth. These are causing sensitivity and are approximately 3mm deep.
What is the correct management option?
● Provide tooth brushing instruction and fluoride
● Restore the lesions with compomer
● Restore the lesions with micro-filled composite
● Restore the lesions with a hybrid composite
● Restore the lesions with conventional glass-ionomer
churchil p.126

300 A patient attends with pain of four days duration in a carious upper molar tooth. The pain is
constant and is not relieved by paracetemol. Sleep has been disturbed by the pain. The tooth is
tender to percussion and gives a positive response to Ethyl Chloride.
What is the most likely diagnosis?
● Pericoronitis
● Apical periodontitis
● Marginal periodontitis
● Reversible pulpitis

301 A 14 year old patient attends with a decayed and hypoplastic LL7. A radiograph shows the
presence of an unerupted LL8 and the LL6 is sound.
What would be the most appropriate long-term treatment for this tooth?
● Amalgam restoration
● Antibiotics
● Extraction
D. Root canal therapy
E. Sedative dressing

302 A 30 yr-old patient attends complaining of pain from the lower left quadrant. Clinical
examination reveals ​an extensively restored dentition ​with generally good oral hygiene. There is
no significant periodontal pocketing other:.than an isolated defect in the region of the furcation
of lower left first molar which is non-mobile. The gingival tissue in thiS area appears
erythematous and slightly hyperplastic with a purulent exudate on probing.
From the list below, which is the most appropriate next step?
● Obtain a radiograph
● Biopsy the gingival tissue
● Remove the restoration
● Vitality testing
● Prescribe antibiotics
Vitality testing.
We want to reach a diagnosis as to whether the lesion is endodontic or periodontal. The radiograph may show a
radioloucency in the furcal area in both cases (Primary endo lesions can cause furcal or periapical radiolucency.
Furcation involvement also causes furcal radiolucency) but will still not give us a definite diagnosis. We initially need
to know whether the tooth is vital or not, then carry on with radiographic investigation.

303 A 40 yr old patient had root-canal treatment to his upper first molar. This was performed 6
months ago using contemporary techniques under rubber dam and was crowned after
completion of treatment. He attends complaining of continued discomfort
from this tooth. Radiographic examination shows each of the three roots to be obturated with a
well-condensed filling to the full working length though there is no evidence of in-fill of the
periapical lesion when compared to the pre-op view. What is the most
likely cause of the continued problem?
● Extra-radicular infection
● Contamination of canal(s) with E.faecalis
● Uninstrumented canal​ >​MB 2
● VerticaL root fracture
● Perio-endo problem
RCT was done properly under rubber dam, tooth was crowned, all 3 canals are filled properly, no furcal involvement.
Crestal bone loss which means only a mild bone loss. So everything is ok now however it is upper first molar which
usually has 4 canals so chances are that one canal has been missed.

304 A 21 year old female presents for the first time to your practice. She is very upset with the
appearance of her upper left central incisor. On examination you find healthy oral hard and soft
tissues and excellent oral hygiene. On close examination you can see
that the upper left central incisor is slightly greyer than the upper right central incisor and has a
composite restoration placed palatally.
What is the most appropriate form of treatment given the information you have?
● Bleaching with carbamide peroxide in custom formed trays of upper and lower arches
● A bonded crown
● A composite veneer
● A porcelain veneer
● E. Non-vital bleaching with carbamide peroxide

305 EDTA (ethylene diamine tetra-acetate) has useful roles in certain situations in clinical
dentistry. When would you use EDTA?
A.As a root end filling material
B.As a pulp capping agent
C. As root canal ​chelating agent
D. As a mouthwash
E. As a dentine bonding agent

306 A patient presents with a history of clicking from their temporomandibular joint. This click
occurs mid way through the opening cycle and is consistent. There is some pre-auricular pain
and the lateral pterygoid muscle on the affected side is tender to resisted
movement test. There is no trismus and the click is not present when the patient opens from an
incisor edge to edge relationship, instead of her normal Class I occlusion. The patient would like
treatment.
The most appropriate occlusal splint for this patient would be:
● Stabilisation splint
● Localised Occlusal Interference Splint
● Bite Raiser
● Soft Bite Guard
● Anterior Repositioner Splint
The patient has anterior disc displacement with reduction. that's y he has midway click .. when the condyle tries to
translate.. an anterior repositioner splint will put the condyle in a downword and anterior position allowing the disc to
get back to original position and hea

A.Stabilization Splint: mainly used for the correction of the occlsuion or creating a perfect occlusion for patient with
TMD reflected from occlusal interference, till the muscles return to their normal tension and lengh, then restorative or
selective grinding work will b done
B. Localized occusal splints: used for clenching and bruxism, focusing the occlusion on less number of teeth, making
their propioseptive response sensetive for biting, so the brain avoid excessive biting and then decrease the Bruxism
C.Bite Raiser: used for the raisong of the bite to relive muscle stress mainly
D. Soft Bite Guard: first line of treatment in symptomatic bruxer, act to decrease muscle spasm and decrease the
habbit.
E. Anterior repositioning splint: it is done as we say [to catch up the disc] as the patient doesnt experience clicking
when he opens from Edge to Edge, thats means that first stage of translation is executed in this edge to edge
movement where the condyle and the disc are aligned as in normal position. keeping this realtion for a long time
allows the retrodiscal tissues [where pain is moderate ] to heal on this level and the pterigoid spasm will b decreased.
THAT`S THE ANSWER TO THE CASE
BEST REGARDS
EBTISAM

Combination muscle and disc disorders are identified by joints that click or pop, and muscle symptoms are also
present. These disorders tend to be more chronic in nature (unless there has been an acute exacerbation), and are
associated with more damage. Stabilization splints are the treatment of choice, as they provide long-term wear that is
usually needed. They also cover the entire dental arch, ensuring that the covered teeth do not move. They must be
worn continually for 24 hours (except when eating) for as long as required to eliminate muscle, disc, ligament, and
tooth symptoms. Three to 6 months of wear is often required. These disorders may be reversible if detected relatively
early and treated appropriately.
Types of Splints:

Bite raising planes​- for bruxists or muscle pain due to hyperocclusion grinding or clenching to free occlusion (
simple mfpd)

Stabilization splints​-for temporomandibular pain dysfunction (tmpds) with muscle tenderness and joint clicking

Anterior repositioning splints-


For ddd (Disc displacement disorder). And acute trauma to keep condyle away from retrodiskal tissue and minimuze
inflammation

DD of tmpds and ddd:


Wenever dey say reproducible click or reciprocal click its ddd wid red
Wen locked jaw in absence or presence of click its ddd widout red
Muscle tenderness plus intermitent click is tmpds

307 A patient presents with a history of a post-crown having fallen out. The post-crown was
originally placed fifteen years ago and had been successful up until four months ago since when
it has come out and been recemented four times. At recementation there
was no evidence of any caries. The patient had been a regular attender and not needed any
restorative treatment for the last eight years.
Which of the following is the most likely cause for the failure of this crown?
● The post was to narrow
● The post was to short.
● The root canal treatment was failing.
● A vertical root fracture was present.
● There were excessive occlusal loads on the tooth.

308 A 23 year old patient attends complaining of pain in an upper right molar and is keen to
keep the tooth. The pain is typically sharp in nature, is triggered by cold and persists after
removal of the cold stimulus. The tooth is not tender to percussion; a
radiograph of the upper right first molar shows a large radiolucency extending to the pulp horn
but no peri-radicular changes.
What treatment is most likely needed in this case?
● Oral pygiene instruction and fluoride application
● Excavation of caries and placement of a permanent restoration
● Root -ca na I treatment?
● Indirect pulp cap and restoration?
● Direct pulp cap and restoration

309 A patient attends your surgery complaining of severe pain, swelling and mobility associated
with a lower first molar tooth in which there is a broken filling. A periapical radiograph indicates
that the tooth has not been root filled and there is loss of apical lamina dura associated with the
distal root and at the bifurcation. The periodontal bone support is good. There is no significant
pocketing.
What is the likely diagnosis?
● Chronic periapical periodontitis
● An acute periodontal-endodontic periodontitis
● A chronic periodontal-endodontic periodontitis
● An acute periapical periodontitis
● Pararadicular periodontitis

310 An upper incisor in a 16 year old patient has suffered trauma and the coronal tissue has
been lost. The tooth has been endodontically treated.
How is the tooth best restored?
● With a fibre post, direct core and crown.
● With a direct core and crown.
● With composite
● With an indirect post-core from a pre-fabricated pattern and a crown
● With a custom indirect post-core and crown

311 A 50 year old male patient has a Class III jaw relationship with an anterior open bite. It is
planned to restore his lower right second molar, which has suffered tooth wear and fracture,
with an indirect restoration. This tooth has approximately 2mm of coronal height.
What would be the most suitable approach to restore this tooth?
● Provide an adhesively retained gold onlay
● Provide a conventional full crown
● Increase the vertical dimension and provide a full crown
● Surgically crown lengthen and provide a gold crown
● Provide an adhesively retained ceramic onlay

312 A patient complains of a lower incisor which has been mobile for several months. The
radiograph indicates a normal level of bony support although the periodontal space has
widened. The apical bone appears normal. The tooth is tender to pressure.
Which of the following tests and or examinations would be most likely to provide a diagnosis?
● Masticatory muscle palpation
● Electric pulp test
● Occlusal examination
● Ethyl chloride test
● Hot gutta percha application
occlusal trauma
313 A 23 year old male presents to your surgery. He lost his upper lateral incisors some 10
years ago in a swimming pool accident. Since then he has been wearing a 'spoon' denture
which he now feels in aesthetically unacceptable. He has sought an opinion on
dental implants but has been told that he would need bone grafting for this to be successful and
he is not prepared to undergo this. His dentition is excellent with no restorations and a Class I
occlusion. He wants some advice on what the best treatment might be.
Which option would you put first on your list of possibilities?
● Two fixed - fixed resin bonded bridges using the central and canine teeth
● Two cantilever resin bonded bridges from canines.
● Two conventional fixed - fixed bridges from the canine
● Conventional cantilever bridges from the canines
● Cobalt chrome partial denture

314 A stabilisation splint [Michegan splint] is commonly indicated in patients needing advanced
restorative dentistry and patients suffering from some Temporomandibular Disorders
Other than upper and lower impressions, which of the following records will be needed to
construct this splint?
● Facebow and a protrusive wax record
● Facebow and an "Oclusal Sketch"
● Centric Relation [Retruded Contact Position] record
● Facebow and a Centric Occlusion [Incuspation Position] record
● Facebow and Centric Relation [Retruded Contact Position] record _

315 A 55 year old female patient is missing her upper right second premolar and upper right first
molar and also is missing the upper left second molar. The upper right second molar is
functional and has an amalgam restoration (MOD and buccal wall) that
requires replacing. The patient has no functional or aesthetic concerns.
What would be the treatment of choice in this situation?
● Provide an upper removable partial denture
● Replace the amalgam in the upper right 7 only >​bcs patient has no funct and esth concerns
● Provide a full coverage crown in the upper right 7
● Provide a fixed bridge in the upper right quadrant
● Provide a full coverage crown in the upper right 7 with guide planes and occlusal rests

316 Endodontic treatment has failed on an upper first molar; the patient is keen to retain the
tooth. There are persistent symptoms from the tooth. Radiographically there is evidence of
periapical radiolucency although the three canals are obturated with good length and
compaction.
What is the best course of action?
● Extract the tooth
● Re-treatment with an iodine solution as irrigant
● Re-treatment with hypochlorite solution as irrigant
● Re-treat the tooth, looking particularly for additional canals
● Surgical apicoectomy

317 You suspect that there is occlusal caries in the lower right first permanent molar of a 10
year old child. You wish to confirm your suspicions.
Which diagnostic test is most commonly used in this situation?
● Bitewing radiography
● Electro-conductive caries monitors
● Fibro-optic transillumination
● Panoramic radiography
● Visual examination of a dried tooth.

318 You notice that a 20 year old patient has marked tooth surface loss associated with the
labial and palatal aspects and incisal edges of the upper anterior teeth. They are sensitive to hot
and cold. The remainder of the dentition is mainly unaffected
What is the likely diagnosis?
● Attrition
● Active erosion >sensitivity
● Passive erosion
● Abfractions
● Abrasion

319 The best radiograph for investigating the maxillary sinus is,
● Periapical radiograph
● Panoramic view
● Lateral cephaloghraph
● Occipitomental view
● Reverse Towne's view

320 Which is true in regard to osseointegration of implants in dentistry?


● The osseointegration is directly between titanium and bone
● Following insertion, implants can be immediately loaded without problem
● The success of the implants is directly proportional to its area of contact with bone and
the bond is mechanical in nature
● The success of the implants depends mostly on low torque preparation and insertion of
the fixture
● The success of integration is accurately investigated by immediate radiographic
examination
321 In the preparation of Premolar class I cavity what is the best way of getting retention,
● Slightly done undercut of the mesial and distal walls
● Slightly done undercut ofthe buccal and lingual walls
● The convergence of the cavity walls

322. In respect to Class V


● it occurs on the buccal groove (fissure)
● it occurs on the lingual groove (fissure)
r>; C. it is a result of bad oral hygiene

323. After the placement of a class I amalgam the patient comes back to you complaining of
pain on masticating and biting; what is
the first thing you would look at,
● Occlusal height
● Contacts areas

324. Child comes to your clinic with a fractured incisor 3 mm super-gingival, how would you treat
the case,
A~ Formocretasol pulpotomy
● Calcium hydroxide pulpotomy >cveck
● Pulpectomy
● Direct capping
● Indirect capping

325. What is Ante's Law about,


● The relation between the span of the bridge and the poetics
● The periodontal area of the abutment teeth
● The relation between the length of the root and the abutment.

326. What is the best way to cement Maryland bridge,


A.GIC
● Resin
● High compression restorative resin
● Zinc Phosphate cement
● Oxide Zinc and eugenol
327. The ideal length of ​core​ in the fabrication of crown and core of endodontically treated tooth
is,
● 1.5 of crown length > core not post
● The length of the crown
● 2/3 tooth/root length
● D. 1/2, root length
Its 2/3 the root length or same as anticipated crown length

328. If aesthetic is not a concern what is the first thing to do to treat soreness under dentures,
● Take the denture off for a week
● Rinse the denture in nystatin
● Apply tissue conditioner

329. While you finishing a class I cavity, the enamel is sound but you noticed in the dentine and
on the Dento-enamel junction a brown line, what is your response,
A. You leave it and complete the final restoration
B.You extend you preparation and clean it
C.You apply a cover of varnish

330. Dental caries of the proximal surfaces are usually start at,
● Somewhere between the ridge and the contact area
● Just gingival to contact areas
● Just about the gingival margin

331. What is the property of high copper amalgam


● Reduced physical creep
● Higher retention

332. In regards to colours what is Chroma stands for,


● Degree of saturation of hue
● Brightness
● Value
● Contrast
333. Frankfort plane extends from,
● horizontally from Sella to nasion
● Sagittal from
● Horizontally from point on superior aspect of external auditory meatus to orbitale

334. Which of the following local anaesthetic is indicated in case of the need to long acting one
after a surgical operation,
/~ A. Lidocaine
● Mepivacaine
● Bupivacaine (Marcaine)

335. In respect to Lidocaine 2% with 1:100000 vasoconstrictor,


● The toxic threshold is 22ml
● 8.8 ml is the maximum you can give in one session

336. Which one of this restorative method will be LEAST compromised by a core,
● Amalgam
● Composite
● GIC
● Cast gold

337. In preparing a very small proximal amalgam cavity on a molar tooth what would consider,
● Extend the cavity to the gingival margin
● Extend the cavity beyond the contacts areas
● Achieve at least 2mm in dentine
● Extend cavity just beyond dento enamel junction
Beyond c​ontact areas most imp..shuld NOT impede wid contacts during function to avoid sliding contacts phenomena
and avoid undue occlusal generater stresses

338. What is true about partial dentures,


● They cause an immediate changes in the oral plaque behaviour
● Night wearing of dentures reduces plaque accumulation
● Relieving the gingival area reduces the gingival enlargement.

339. The biting load of denture base to tissues compared to teeth are,
● Ten times more
● Ten times less
● Equal
340. Compound is,
● Very accurate compression material
● Thermoplastic material

341. The different between normal stone and the dye stone is,
● In the particles size
● The amount of water

342. A patient comes to you with medium pain of tooth filled with Composite resin as a result of
cold or hot drinks, what your initial management will be,
● Remove the restorative material and start an Endontic treatment
● Remove the restorative material and place a sedative temporary material
● Place a coat of bonding material on the old composite

343. Throbbing pain increases with heat and cold stimuli, the MOST probable diagnosis is,
● Cyst
● Occlusal trauma
● Advanced pulpitis

344. In making your custom trays which of the following is true,


● A uniform thickness is required
● Perforation is better
● Only adhesive is better than perforation

345. The most common cause of caries in children is,


● Soft diet
● High intake of carbohydrate
● Poor oral hygiene

346. The best storage media for avulsed tooth is,


● Saline
● Milk
● Water
● Saliva

347. An occlusal approaching clasp TIP,


● Should occupy a predetermined undercut
● Contact the tooth under the survey line
● Rigid

348. In the construction of partial denture the surveyor is not used to,
● Contour the wax as part of the fabrication of the working cast
● Locate the guide planes
● Determine the location of indirect retainers
● Identify any undesirable undercuts
Surveyor can
-outline desirable and undesirable undercuts
-establish guide planes
-contour wax
-block out undercuts
-establish path of insertion
-check parallelism

But it cannot determine indirect retention or reciprocation.

349. The main purpose of finishing the enamel walls is,


● Remove loose enamel rods
● Provide a better surface for the adoption of restorative material

350. A female patient comes to you complaining of persistent pain in heavily restored central
incisor; you suspect pulpitis and you have been told that she is in transit leaving by plane next
day. Your treatment will be,
● Remove filling and place a sedative dressings
● Pulp extirpation and obturate with Ledermix dressings
● Prescribe analgesics and systemic antibiotic

351.which of the following is true regarding TMJ dysfunction,


● It is always due to arthritis, should be treated with NSAIDS before attempting surgery
● Raising bite increases the space in the joint and should be attempted before surgery
● It is mostly due to the medial movement of the condylar head over the glenoid fossa

352.The location of Class V is in,


● The bucca I pit /fissu re
● The occlusal surface
● The cervical third

353. Occlusal cavity with extension of the buccal fissure is classified as,
● Class II
● Class III
● Class I

354. Which of the following does not affect the elasticity of retentive clasp?
● Length of the arm
● The cross section shape
● The material used
● The undercut area

355. Following calcium hydroxide pulpotomy, the dentist would expect dentine bridge to form at,
● The exact level of amputation
● Level some where below the amputation
● Half way between amputation and apex
● At the apical region of the tooth

356. In the construction of a full veneer gold crown, future recession of gingival tissue can be
prevented or at least minimised by,
● Extension of the crown 1 mm under the gingival crevice
● Reproduction of normal tooth incline in the gingival one third of the crown
● Slight over contouring of the tooth in the gingival one fifth of the crown
● Slight under contouring of the tooth in the gingival one fifth of the crown

357. A partial denture that seats on the master cast but fails to seat correctly in the mouth is a
result of,
● Contraction of the metal framework during casting
● Insufficient expansion of the investment material
● Distortion of impression
● Failure to block out unwanted undercuts
358. Which of the following muscles may affect the borders of mandibular complete denture?
● Mentalis
● Lateral pterygoid

r>. C. Orbicularis oris


● Levator oris
● Temporalis
Coz all muscle in orbacularis oris doesnt affect mandibular denture its mentslis,incisius,buccinator,depressor snguli
oris anteriorly n posteriorly masseter,superior constructor n buccinator lingually ,and geniohyoid n mylohyoid

359. What is correct in regards to high copper amalgam,


● Reacts and strengthens the amalgam by its dispersion properties
● Reacts to form copper-tin phase thereby eliminating the tin-mercury phase
● Reacts to form copper-silver phase thereby eliminating the silver mercury phase
● Reacts and strengthens the amalgam by its grain diffusion

360. What is the main purpose of using corticosteroids in pulpal obturation material?
● For their antibiotic action
● For their antiinflammatory action
● To relief pulp pressure

361. Which of the following statements is incorrect regarding Smoker's Keratosis?


● Typically affects the hard palate
● Minor mucous glands are swollen with red orifices
● There is a little regression if smoking is stopped

● Affects mucosa exposed to smoke (mainly the hard palate) Areas protected by denture
unaffected
● Palate is white (keratotic) with umbilicated swellings with red centres (inflamed mucous
glands)
● Responds rapidly to abstinence from pipe smoking

● Statistically, a raised risk of oral cancer but not in the hyperkeratinised (palatal) area
362. 13 years old boy comes to you with excessive hyperplasia of the gingiva as a result of
Phenytoin what is your management,
● Stop the medication
● Force a strict oral hygiene and surgical removal of excess gingival tissues
● Debridement and conservative approach

363. White man 56 years old comes to you with a brown spot on his gingiva and another one on
his oral mucosa, when taking the history he mentioned a weight and memory lost. He as well
complains of headaches. What is your most probable diagnosis,
● Addison's disease
● Hyperthyroidism

364. While removing the second primary molar of 9 years old child, the apical third of the root
fracture and stay in the socket,
● You will just leave it and observe it
● You take surgically by a lingual flap
● You try to take out by using a root apex elevator
● You use a fine end forceps to take it out

365 What is the most important factor to reduces dental irradiation,


● Speed of film
● Collimation
● Filtration
D. Cone shape and length

366. With view to Nitrous Oxide what is the major pharmacological problem?
● Contraindicated in pregnancy
● Contra indicated in cardiac dysrhythmias > reletive contraindaction

367. Which of the following is an expansile lesion of the oral mucosa,


● Keratocyte
● Radicular cyst
● Cementoma

368. The concentration of Fluoride in the topical NaF


A.2%
B.5%
C. 8%
D.10%

369. patient in your dental chair shows chest pain, weak pulse and dysponea, what is your initial
management,
● Give a nitro-glycerine tablet and keep the patient up seated'
● Put the patient in supine position
● Do nothing and wait until the symptoms go

370. What are two teeth connected at the cementum called,


A. Concrescence
B. Dilaceration
C. Gemination
D. Fusion

371. Developer contaminated with other chemical and was not mixed pro perlv. What is the
effect on the X-ray film?
● Too dark film -
● Light film
● Foggy

372. Which part of the cranium is considered as the most stable area,
● Frankfort plane
● Occlusal plane
● Anterior cranial plane
● Anterior nasal to gnathion.

373.The difficulty of placing matrices on deciduous dentition is a result of,


● The small mouth of kids which result in problem keeping the matrices in their mouths
● The occlusal convergence of the deciduous teeth

374. Branchial Cleft cyst is located,


● Medial to the neck
● On anterior border of the Sternocleidomastoid muscle
● Shows when swallowing

375. the most common way of oral carcinomas to other tissues is/are
● Lymphatic
● Invasion and blood
● Lymphatic and invasion
● Blood and lymphatic

376. The main purpose of periodontal treatment is,


● Elimination of plaque and calculus
● Elimination of periodontal pockets
● Reformation of all the periodontal ligaments
● The elimination of all occlusal trauma
. What is the approximate unstimulated salivary flow rate,
● 2 ml/min
● 0.2 ml/min
● c. 0.02 ml/min D. 20 ml/min

378. The thermal and electric pulp tests will,


● Give an accurate indications of the pulp status
● The patient's response will be either pain or no pain
● The patient can differentiate between cold or hot stimuli

379. Immediately after the extraction of lower molar the patient complains of post operation
bleeding and pain, how would manage this,
● Prescribe analgesics and ask the patient to follow a strict oral hygiene
● Administer 5% Marcaine(​bupivacaine)​ Local Anastatic, prescribe analgesics and pack the
socket with alvogyl
● Administer 5% Marcaine Local Anastatic, suture the socket and prescribe analgesics
● Suture and give pressure packs

380. Gracey curette is characterized by,


● The blade and the shank form 90Q angle
● Can be used on both sides
● Can be used on any tooth surface
● It is specific for each surface of the tooth

381. The removable partial denture requires relining what is would be the most appropriate
action,
● take a new impression by asking the patient to occlude on it
● Provide equal space between denture and gingival tissues.
● Make sure the framework and retainers are seated in place before taking impression

382. In regards to dentine strength, which is the right sequence,


● Affected dentine> Sound dentine> Infected dentine
● Sound dentine> Affected dentine> Infected dentine

383. Symptoms free patient comes to you after four weeks of an endodontic treatment and you
find on radiograph the canal is over filled with what it seems to be a cone of Gutta Percha lmm
beyond the apex with a radiolucent small area. What is your initial
management?,
● Start apiectomy through a flap and surgery
● Obturate the root canal
● Ask for a recall and observe in three months time
● Seal the pulp chamber and keep it under observation
384. After obturation and on X-ray you notice the obturation materials are lmm beyond apex.
What is your first management?
● Refill the canal
● Pull the GP cone about lmm out and take a new X-ray
● Leave it as it

385. 2.21mg NaF contains,


A. lmg fluoride
B.2 mg
C. 0.5 mg

386. Stiffness of material are measured by,


● Proportional unit
● Modules of elasticity
● Stress/ strain

387. Four years kid shows at your clinic with open bite as a result of thumb sucking, you notice
a delayed speech ability what would be your first management,
● Refer to a speech therapist
● Apply a removable habit inhibitor denture
● Apply a removable habit inhibitor denture and educate the parents about it so the kid will
not be taking it off so often

388. Two central incisors on a radiograph are showing with what looks like eye drop
radiolucency. You decided to start endodontic treatment on these teeth but you tried to open
access to the root canal you find clearly closed orifices with what look like secondary dentine.
What is your initial management?
● Leave as it and start a permanent restoration.
● Start systemic antibiotic
● Try to ream and file canals

389. A patient with no positive history came along for scaling. The moment you pick up the
scaler you punch your finger, what should you do?
● Complete the procedure as nothing has happened
● Check patient's blood for Hepatitis B antibody HBsAb
● Check patient's blood for Hepatitis B antigen HBsAg
● Check dentist's blood for Hepatitis B antibody HBsAb and HIV antigen HIVAg
● Check dentist's blood for Hepatitis B antigen HBsAg and HIV antibody HIVAb

390 when probing for periodontal disease the tip of the probe will be,
● At the coronal end of junctional epithelium
● At the top of the gingival calculus

391. After the initial development stage and in the absence of pathology, the size of the pulp
chamber has been reduced by,
● Deposition of primary dentine
● Deposition of secondary dentine
● Reparative dentine
● Pulp fibrosis

392. The most desirable outcome of endodontic treatment is,


● The healing of the alveolar bone
● The deposition of cementum at the apex
● Formation of fibrous capsule around the apex

393. What is NOT related to the normal aging process?


● Progressive bone loss
● Reduced elasticity of muscles
● Decreased elasticity of the skin
● Lower pain threshold

394. The most accurate finding of pulpal pathology


● Radiolucency on the apical region
● Pain on hot or cold drinks
● The absence of response to pulp testing

395 For dental caries to progress in dentine,


● enamel must contain glycoproteins
● diet must contain simple carbohydrate
● diet must contain polysaccharides
● pulp must contain complement
● the dentine must contain soluble collagen

396 A patient on the dental chair has cardiac arrest which is INCORRECT,
● Observing the vital signs and check that the air way is clear is at high importance
● Expired air has 15% 02 only, and cardiac compressions achieve 30-40% of cardiac
output
● Intermittent positive pressure at the rate of 40/min will reduce the chances of cerebral
hypoxia
● Intermittent positive pressure is better than mouth to mouth when it has been given at
the same rate.

397. A 9 year old boy has a small white discolouration on his maxillary central incisor. The
lesion is most probably,
● Hypocalcification due to trauma of the primary predecessor
● Hypoplasia due to acute systemic infection when 6-12 months old
● Defect during the histo differentiation stage of development
● Defect during the morho differentiation stage of development
In hypocalcification of successor due to trauma of predecessor you will see "whitish discoloration" (white spots) - due
to hypocalcification.
&
In hypoplasia due to acute early childhood illness or stressfull birth , you obseve "greyish whitish bands"
(An attenuation of lines of retzius)

398. The best method to take X-ray of the maxillary sinus is,
● Periapical radiograph
● Panoramic view
● Lateral cephaloghraph
● Occipitomental view
● E. Reverse Towne's view

399. Which of the following is not a part of the fully formed enamel organ,
● Outer enamel epithelium
● Inner enamel epithelium

400.18 years old female her weight is 52Kg and she is 163cm tall. On dental examination
erosion of teeth on the most of her lingual surfaces is clearly showing. Dietary history revealed a
daily rate of 5000 to 7000 Kcal/day. What is most probable would be
her case?
● Alcoholism
● Drug abuse
● Bulimia
● Excessive smoking
● Diabetic mellitus type I

401. Which one of the following is true in regards t osseointegration implants in dentistry?
● Fibrous tissues are formed and integrated directly between titanium and bone
● Following insertion, implants can be immediately loaded without problem
● The success of the implants is directly proportional to its area of contact with bone
● The success of the implants depends mostly on low torque preparation and insertion of
the fixture

402. Generalised lost of tooth structure by chemical means called,


● Erosion
● Attrition

403. On X-ray, the buccal roots of 16 is considerably elongated; this is a result of,
● Too great vertical angulation
● Inadequate vertical angulation
● Excessive object film distance

404. The principle muscle responsible for the opening of the mouth is,
● Mylohyoid
● Anterior temporal
● Posterior temporal
I.​ Anterior belly of digastric

405. Denture stomatitis is commonly associated with,


● The continuous wearing of removable orthodontic appliances in otherwise healthy
patient
● The proliferation of hypertrophic tissue at the denture periphery
● The overgrowth of some constituents of oral normal microflora
● Allergy to denture base material

406. Which of the following is NOT characteristic of Down's syndrome?


● Decreased neutrophil function
● Macroglossia
● Macrodontia
● An increased susceptibility to periodontal disease
● Congenitally missing teeth

407. Which of the following is the best evidence that a previous periodontal treatment is
successful?
● The patient keeps a 3 month recall appointment
● There is no extrinsic stain
● The patient demonstrates good understanding of brushing and flossing techniques
● There is no bleeding on probing

408 A 65 year old female presents at your surgery complaining of an extremely sharp pain of a
few seconds duration which arises
whenever she touches an area of skin above the right hand side of her upper lip adjacent to the
angle of her mouth. The patient is
fit and well and is not taking any medication. You make a diagnosis of trigeminal neuralgia.
What is the drug of choice for treating trigeminal neuralgia in such a patient?
● Carbamazepine
● Oxcarbamazepine
● Gabapentin
● Phenytoin
● Ibuprofen

409 A patient on examination was found to have swollen gingiva around a crown that had been
present for several years. The
papillae were particularly enlarged.
What is the most important feature of a crown that may be responsible for this?
● Material of the Crown
● The occlusion
● proximal Contour
● Labial Contour
● Surface finish

410 A 13 year old girl presents with an unerupted permanent canine and a retained primary
canine. You cannot palpate the unerupted canine in the buccal sulcus and you are uncertain as
to whether it is displaced palatally or in the line of the arch.
Which single radiographic view would be most helpful in locating the unerupted tooth?
A. Bitewing
B.DPT
● Lateral oblique
● Single periapical
● Upper anterior occlusal

411 A 30 year old man with unknown allergy to latex goes into anaphylactic shock whilst being
treated in the dental surgery.
Which drug and route of administration is of most benefit in this situation?
● Hydrocortisone - orally
● Chlorphenamine - intramuscularly
● Chlorphenamine - orally
● Epinephrine - intravenously
● Epinephrine - intramuscularly

412 A 62 year old female presents at your surgery complaining of a persistent, dull ache
affecting her upper left 4. The pain is present all the time but varies in its severity although the
patient cannot think of any exacerbating factors. It is not relieved by analgesics. Over the past
18 months the patient has had several teeth extracted from the upper left quadrant. Each
extraction brings about temporary relief of her symptoms only for them to recur in an adjacent
tooth.
What is the most likely cause of the patient's pain?
● Acute/reversible pulpitis
● Dentine sensitivity
● Chronic/irreversible pulpitis
● Atypical odontalgia
● E. Trigeminal neuralgia

413 A 25 year old patient attends your surgery complaining of a swelling at the angle of the
mandible. A radiograph shows a uni-locular radiolucency associated with the crown of an
unerupted wisdom tooth.
Which of the following is the most likely diagnosis?
● Radicular cyst
● Dentigerous cyst
● Lateral periodontal cyst
● Ameloblastoma
● Odontogenic keratocyst
Ameloblastoma and keratocyst are usually multilocular.asscoiated with diplacement of tooth/roots.
When its unilocular surrounding unerupted crown -always dentigerous)

414 A 70 year old female, who suffers with persistent looseness of her lower complete denture,
is considered for implants. She will require radiological evaluation of the potential implant sites.
Which would be the most appropriate radiological investigation at this stage?
● MRI scan of the mandible
● CT scan of the mandible > first choice is cbct then ct
● Periapical radiographs of the mandibular anterior region
● True lower anterior occlusal view
● Panoramic radiograph
Cone beam CT. This is ideal for implant assessment and is likely to become the imaging modality of
choice when cross-sectional imaging is required .

415 Six months ago you saw a child patient, then aged 9 years. His upper right maxillary canine
was palpable in the labial sulcus but the upper left was not. The situation is now unchanged, so
you have taken two periapicals of the non-palpable tooth. They both show that there is some
resorption of the CI root but the permanent canine appears somewhat mesially angled and is
more mesial on the more mesially positioned film.
What is your the best course of action?
● Keep a careful watch on it and take another x-ray in 6 months.
● Refer to an oral surgeon for early exposure of the permanent canine.
● Refer to an oral surgeon for early removal of the permanent canine before it damages
the lateral incisor.
● Refer to an orthodontist for a treatment plan.
● Wait and watch' until the child is 11.

416 An apical radiolucency (2mm) is noticed as an incidental radiographic finding associated


with the apex of the mesiobuccal root of the lower right first molar. The tooth has been root filled
but is 2mm short of the radiographic apex. There are no other clinical or radiographic findings
and the patient is fit and well. What is the most appropriate course of action?
A.Extract the tooth.
b.Redo the root filling
c.Perform periapical surgery.
d.Advise the patient of the situation and monitor clinically and radiographically
e.Prescribe antibiotics then review

417 A 43year old patient is missing on the upper right the first premolar and molar. He has good
oral hygiene and requests a fixed replacement for these teeth. The other teeth on the same side
are all moderately restored with MOD amalgam restorations and are vital, except the canine,
which has a very large restoration and is root-filled. He has group function. Radiographs show a
large sinus cavity and no peri-apical pathology. What would be the restoration of choice for
replacement of the missing teeth?
● Implant supported crowns
● A conventional fixed bridge using the 7 and 5 as abutments
● Two conventional cantilevered bridges, using the 7 and 3 as abutments
● A resin-bonded bridge, using the 7 and 5 as abutments
● A conventional fixed-moveable bridge using the 7 and 5 as abutments

418 Epidemiological studies have shown that dental decay is normally greater amongst
northerners and those in socially deprived circumstances. Which of the following would be most
effective in reducing cariesJn a high risk population?
● Brushing with fluoride toothpaste
● Publicity campaign
● Fluoridation of the water supply
● Dietary advice via schools
● Fissure sealant provision

419 A 58 year old male presents at your surgery complaining of a sharp pain of no more than 30
minutes duration arising from his upper left molar region. The pain is brought on by cold stimuli
but persists after-the stimulus is removed. It does not seem to occur spontaneously. He has
tried taking paracetemol and this does temporarily stop the pain from recurring. The upper left 6'
reacted to a lower current on electronic pulp testing than the upper right 6, upper left ~ or the
lower left molars.
What is the most likely cause of the patient's pain? -
● Acute/reversible pulpitis
● Dentine sensitivity
● Chronic/irreversible pulpitis
● Periapical periodontitis
● Trigeminal neuralgia
420 A 60 year old patient attends your surgery complaining of a sore mouth. He has Type II
diabetes well controlled by diet and metformin. On examination white patches which cannot be
removed are present on his buccal mucosa:
What is the most likely diagnosis?
● Frictional keratosis
● Leukoplakia
● Lichen Planus
● White sponge naevus
● Candidosis

421 A 60 year old female attends your surgery complaining of soreness affecting her gingivae.
No other area of her oral mucosa is affected but she has noticed an itchy rash on the flexor
surface of her forearms. She is fit and well and is not taking any medication. Scattered
purple/red papules each about 4mm in greatest dimension are present on the flexor surface of
her forearms and on intraoral examination a desquamative gingivitis is present. Based on the
above findings what is your diagnosis?
● Mucous membrane pemphigoid
● Lichen planus
● Pemphigus vulgaris
● Erythema multiforme
● Lichenoid drug reaction
pink

422 An 80 year old patient presents with an ulcer in the floor of the mouth. This has been
present for several months and has not responded to conventional treatment. An incisional
biopsy is taken. Which of the following histological changes in the epithelium
confirm a diagnosis of squamous cell carcinoma?
● Hyperkeratosis
● Acanthosis
● Dysplasia
● Invasion
● Discontinuous epithelium
423 A 35 year old patient complains of swollen gums. This has been present for several years.
What is the commonest cause of this complaint?
● Cyclosporin therapy
● Vitamin C deficiency
● Chronic Gingivitis
● Atenolol therapy
● E. Pregnancy

424 A 45 year old patient attends the dental clinic complaining of a clicking jaw. Examination
reveals a reproducible click of the right TMJ when opening wide. Upon asking the patient to
open wide, close with incisors edge-to-edge and then open and close to
this position, the click is absent. From the options below, which one is the most likely diagnosis?
● Myofascial pain
● Disc displacement with reduction
● Disc displacement without reduction
● TMJ osteoarthritis
● Arthralgia
425 Cigarette smoking is considered to be the most important factor next to microbial plaque in
periodontal disease progression.
Which of the following is the most important factor in the disease progression in smokers?
● Smokers have drier mouths than non-smokers .,
● Smokers have poorer oral hygiene than non-smokers.
● Nicotine will impair the chemotactic and phagocytic properties of PMNs.
● The gingival blood flow is reduced in smokers.
● Smokers alter the oral environment encouraging the growth of anaerobic bacteria
426 An adult patient attends your practice complaining of pain and swelling associated with a
previously restored upper first premolar tooth. The pain has been present for a number of days
and is no longer responding to analgesics. His dentition is otherwise well maintained and his
periodontal health is good.
What is the most appropriate approach to treatment?
● Antibiotics and analgesics.
● Extract the tooth
● Carry out a pulpotomy. Temporary dressing.
● Carry out a pulpectomy. Temporary dressing
● E. Establish open drainage

427 A patient reports that his post crown has fallen out. This crown had been present for many
years. You note that there appears to be a hairline vertical fracture of the root. The tooth is
symptomless.
What is the most sensible approach to treatment?
● Replace the post crown using a resin-reinforced glass ionomer material
● Replace the post crown using a polycarboxylate cement
● Replace the post crown using a dentine bonding agent and a resin- reinforced glass
ionomer material
● Replace the post crown using a resin composite luting agent
● Arrange to extract the tooth

428 A patient says that he does not like the appearance of his previously root filled upper
central incisor tooth. His dentition is otherwise well maintained and his periodontal health is
good. The tooth appears to be darker than the adjacent teeth.
What is the most appropriate approach to treatment?
● Provision of a post crown
● Provision of an all ceramic crown
● Provision of a metal bonded to ceramic crown
● Carry out a non vital bleaching procedure
● Provision of a porcelain veneer

429 A 20 year old patient attends your surgery for the first time. You suspect that he may have
proximal caries as he has a frequent sugar intake. Which of the following is the most accurate
method of diagnosing proximal caries in a lower molar tooth in this young
adult?
A. Clinical history
● Periapical radiograph
● Bitewing radiograph X
● Digital image
● Electronic resistance measurements
430 A 25 year old male attends for the first time complaining of sensitivity of a number of teeth.
On examination, the occlusal surfaces of all the teeth are worn with obvious wear facets on the
canines and premolars. Posterior amalgam restorations are proud of the surrounding tooth.
What would be the first stage management?
● Take impressions for study models
● Prescribe fluoride mouth rinse
● Replace the amalgam restorations
● Dietary analysis
● Placement of resin sealant to sensitive teeth

431 A 35 year old male patient who admits to grinding his teeth at night has a number of wedge-
shaped cervical (Class V) lesions on his upper premolar teeth. These are causing some
sensitivity and are approximately 3mm deep. What is the correct
management option?
● Provide tooth brushing instruction and fluoride
● Restore the lesions with compomer
● Restore the lesions with micro-filled composite
● Restore the lesions with a hybrid composite
E. Restore the lesions with conventional glass-ionomer

432 A patient attends with pain of four days duration in a carious upper molar tooth. The pain is
constant and is not relieved by paracetemol. Sleep has been disturbed by the pain. The tooth is
tender to percussion and gives a positive response to Ethyl Chloride. What is the most likely
diagnosis? .,
● Pericoronitis
● Apical periodontitis
● Marginal periodontitis
● Reversible pulpitis
● Irreversible pulpitis

433 You are trying in a partial chrome denture framework which fails to seat properlv. It fits the
master cast.
What is the most likely cause of this problem?
● Insufficient expansion of the investment material
● Distortion of the impression
● Contraction of the metal framework during casting
● Failure to block out unwanted undercuts
● Complex denture design
434 Bitewing radiography is the main special text used to help in diagnosis of proximal caries.
The performance (accuracy) of a diagnostic test like bitewing radiography can be expressed in
terms of sensitivity and specificity.
Which of the following is a reasonable summary of the diagnostic accuracy of bitewing
radiography for proximal caries diagnosis?
● Moderate sensitivity and low specificity
● Moderate sensitivity and moderate specificity
● Moderate sensitivity and high specificity
● D. High sensitivity and moderate specificity
● E. High sensitivity and high specificity

periapical has Moderate sensitivity and high specificity

435 You are interested in finding out what the risk indicators are for a rare form of oral cancer
and decide to undertake a study to examine this.
What type of study would be the most appropriate for addressing this issue?
● Cohort
● Prevalence study
● Clinical trial
● Case-control study

436 You take a panoramic radiograph of a patient and discover a well-defined, corticated
radiolucent area below the inferior dental canal just anterior to the mandibular angle. What is the
most likely diagnosis?
● Radicular cyst
● Stafne bone cavity
● Metastatic carcinoma of the breast
● Adenomatoid odontogenic tumour
● Complex odontome

437 You take a panoramic radiograph out of a patient's records but you find that the film has a
low density and poor contrast.
Which of the following errors could lead to low density and poor contrast?
● Too long a development time > poor contrast dark film or high density
● Developer temperature too high >poor contrast dark film
● Developer temperature too low> poor contrast pale or low density film
● Inadequate fixation > poor contrast
● poor storage > poor contrast

438 You want to evaluate the effectiveness of using tetracycline as an adjunct to scaling and
root planing for the treatment of chronic periodontitis. What type of primary study design would
be most appropriate for addressing this topic?
● Cohort study
● Non-randomised controlled trial
● Randomised controlled trial
● Case-control study E. Case-series

439 Radiation protection of patients is partly dependent upon equipment factors (x- ray set and
film or digital system). The different factors that can be changed vary in financial cost to the
dentist and in their effectiveness in cutting x-ray dose.
Which of the following provides the most cost-effective means of minimizing patient radiation
dose in dental intraoral radiography?
● Constant potential ('DC) x-ray set
● Rare earth filtration
● Lead apron
● D speed film
● F speed film

440 A 42 year old man presented with a firm fixed swelling in the right preauricular region which
had been enlarging over the last month. He had also developed a right sided facial palsy.
What is the most likely diagnosis?
● Adenoid cystic carcinoma
● Squamous carcinoma
● Sebaceous cyst
● Branchial cyst
● Pleomophic adenoma

441 A 14 year old patient attends with a decayed and extensive hypoplastic LL7: He is a very
irregular attender with poor oral health habits. A radiograph shows the presence of an
unerupted LL8 and the LL6 is sound.
What would be the most appropriate long-term treatment for this tooth?
● Amalgam restoration
● Antibiotics
● Extraction
● Root canal therapy
● Sedative dressing

442 A 7 year old boy has previously had all primary molars restored and a pulpotomy on upper
right E. He has an early mixed dentition with lower lateral incisors erupting. There is a midline
diastema of 2 mm. The upper right E has become symptomatic and
requires extraction.The most likely long term effect of the extraction on the occlusion is:
● Early eruption of the second premolar.
● Loss of upper central line.
● No significant effect.
● Overeruption of the lower right teeth.
E. Potential crowding in the upper right quadran​t

443 Which of the following is the ideal treatment for a degree II furcation involvement of a
mandibular molar?
● Tunnel preparation
● Root resection
● Furcation plasty
● Extraction
● Guided Tissue Regeneration

444 You examine a patlent and find BPE code 4 in all sextants. Radiographs show generalised
horizontal bone loss with a minimum of 50% of bone support remaining on all teeth.
Which of the following is the most important factor when considering the prognosis for the
teeth?
● Age of the patient
● The Oral Hygiene Status
● Bleeding on Probing Score
● Mobility
● Gingival Recession

445 A 30 yr-old patient attends complaining of occasional pain from the lower left quadrant.
Clinical examination reveals an extensively restored dentition with generally good oral hygiene.
There is no significant periodontal pocketing other than an isolated defect in the region of the
furcation of lower left first molar which is non-mobile. The gingival tissue in this area appears
erythematous and slightly hyperplastic with a purulent exudate on probing.
From the list below, which is the most appropriate 'next step?
A. Obtain a radiograph
● Biopsy the gingival tissue
● Remove the restoration
● Vitality testing
● Prescribe antibiotics

446 A 40 yr old patient had root-canal treatment to his upper first molar. This was performed 6
months ago using contemporary techniques under rubber dam and was crowned after
completion of treatment. He attends complaining of continued discomfort
from this tooth. Radiographic examination shows each of the three roots to be obturated with a
well-condensed filling to the full
working length though there is no evidence of in-fill of the periapical lesion when compared to
the pre-op view. There is crestal
bone loss and no furcal involvement. What is the most likely cause of the continued problem?
● Extra-radicular infection
● Contamination of canal(s) with E.faecalis
● Uninstrumented canal
● Vertical root fracture E. Perio-endo problem

447 A mother is concerned that her child's adult upper front teeth have not erupted and asks
your advice.
What is the usual age in years for the upper permanent central incisors to erupt?
● 5 years.
● 6 years.
● 7 years.
● 8 years.
● 9 years.

448 You decide to refer an eight year old child to the oral surgery department in your local
hospital for extractions under a general anaesthetic. What key reason for asking for a general
anaesthetic would you put in the referral letter?
● Parents request GA.
● Failed to complete treatment under inhalation sedation.
● Child would not accept local analgesia
● Parents think hospital more convenient.
● Not one of my regular patients.

Circumstances and conditions which rarely justify GA.


● Carious, asymptomatic teeth with no clinical or radiographic signs of sepsis.
● Orthodontic extraction of sound permanent premolar teeth in a healthy child.
● Patient/carer preference, except where other techniques have already been tried.

● Extenuating circumstances that override the above limitations are:


● Physical, emotional, learning impairment or a combination of two or more
of these.
● Children who have attempted treatment using LA alone or LA combined
with conscious sedation and been unable to co-operate.
● Medical problems which are better controlled with the use of GA.

449 An eight-year-old boy presents with pain of three days duration that has kept him awake.
On examination you see a grossly carious lower left 6 and some associated buccal swelling.
Which of the following is the most appropriate to give immediate relief of his pain?
● Extract the LL6.
● Gently excavate the.cartes and obtain drainage.
● Give antibiotics.
● Incise any swelling
● Refer for-general anaesthetic
450 A 10 year old girl tripped and-knocked out her upper incisor 10 minutes ago. She is holding
it in her hand. What is your most effective immediate treatment?
● Accept the tooth is lost and do nothing.
● Refer to specialist.
● Re-plant immediately.
● Root fill, clean and re-plant.
● Sterilise root in alcohol and re-implant.

451 You are treating a patient who has a long history of recurrent episodes of oral candidosis.
His mouth has recently become sore once again and you want to prescribe some antifungals for
him. You check his medical history and find that he suffers from atrial fibrillation and is taking
warfarin.
Which of the following drugs would be most appropriate to treat this patient's candidosis?
● Metronidazole
● Nystatin
● Fluconazole
● Miconazole
● Amoxycillin

452 A dental company has claimed that ozone is better than conventional methods for treating
decay in peoples' mouths.
Which of the following would provide t​ he best evidence to support these claims?
● Systematic review of Randomised Controlled Trials
● Randomised controlled trial
● Cohort study
● Case/control study

453 A GDP in Manchester randomly selects 5000 patients, aged between 45 and 55 years, who
are free from periodontitis. He determines that 300 of them are smokers. He follows all 5000
patients for 10 years, by which time 150 had developed periodontitis. Of the patients with
periodontitis, 60 had previously been identified as smokers and 90 as non-smokers.
What type of study is this?
● Case-series
● Prevalence study
● Clinical trial
● Case-control study
● Cohort

454 Randomised controlled trials enable us to establish cause and effect.


The main reason for using this study design is to:
● Ensure that making a Type II error is minimised.
● Ensure that test and control groups are similar.
● Ensure that test and control groups are of equal size.
● Help recruitment of patients into the trial.
● Provide sufficient power for the study.

455 A 53 year old male patient presents with an asymptomatic white patch on the ventral
surface of his tongue. He has smoked 35 cigarettes a day since he was 17. An incisional biopsy
of the white patch is likely to show which histological feature:
● Basal cell liquefaction
● Hyperkeratosis
● Saw tooth rete ridges
● Acantholysis
E. Basal cell hypoplasia

456 A 46 year old female presents with a slowly enlarging painless firm swelling in the hard
palate to the left of the midline.
The most likely diagnosis is:
● A dental abscess
● Torus palatinus
● Osteoma
● Pleomorphic adenoma
● Canalicular adenoma

459 A 24 year old man presents with a loculated cystic radiolucency in the lower third molar
area approximately 3cm in diameter. nThere is no tooth associated with the cyst and there is no
bucco-lingual expansion of the mandible. There is no resorption of the
roots of the overlying second molar tooth.
How would this be treated?
● Marsupialisation
● Enucleation and ethyl chloride
● Block resection
● Enucleation and Carnoy's solution >keratocyst
● Enucleation and formalin solution

460 A 21 year old female presents for the first time to your practice. She is very upset with the
appearance of her upper left central incisor. On examination you find healthy oral hard and soft
tissues and excellent oral hygiene. On close examination you can see that the upper left central
incisor is slightly greyer than the upper right central incisor and has a composite restoration
placed palatally. What is the most appropriate form of treatment given the information you
have?
● Bleaching with carbamide peroxide in custom formed trays of upper and lower arches
● A bonded crown
● A composite veneer
● A porcelain veneer
● Non-vital bleaching with carbamide peroxide

461 EDTA (ethylene diamine tetra-acetate) has useful roles in certain situations in clinical
dentistry. When would you use EDTA?
A.As a root end filling material
B. As a pulp capping agent
e. As a root canal chelating agent
● As a mouthwash
● As a dentine bonding agent

462 A patient presents with a history of clicking from their temporomandibular joint. This click
occurs mid way through the opening cycle and is consistent. There is some pre-auricular pain
and the lateral pterygoid muscle on the affected side is tender to resisted movement test. There
is no trismus and the click is not present when the patient opens from an incisor edge to edge
relationship, instead of her normal Class I occlusion. The patient would like treatment.
The most appropriate occlusal splint for this patient would be:
● Stabilisation splint
● Localised Occlusal Interference Splint
● e. Bite Raiser
D. Soft Bite Guard
E. Anterior Repositioner Splint

463 Glass ionomer cements are used in restoring Class V cavities.


Which of the following constituents are most likely to be present in glass ionomer'cements?
A Alumino-silicate glass and phosphoric acid
B. Phosphoric acid and zinc oxide
C.. Polyacrylic acid and zinc oxide
D.Aluminosilicate glass and polyacrylic acid
E.Itaconic acid and zinc oxide

465 A patient presents with a history of a post-crown having fallen out. The post- crown was
originally placed fifteen years ago and had been successful up until four months ago since when
it has come out and been recemented four times. At recementation
there was no evidence of any caries. The patient had been a regular attender and not needed
any restorative treatment for the last
eight years. Which of the following is the most likely cause for the failure of th"fs crown?
● The post was to narrow
● The post was to short.
e. The root canal treatment was failing.
● A vertical root fracture was present.
● There were excessive occlusal loads on the tooth.

466 In dental epidemiology, indices are used to measure the oral health of a population. The
DMF index is commonly used to
measure the prevalence and severity of dental caries.
What is the main limitation of this approach?
● It does not allow statistical analysis.
● It gives equal weight to decayed, missing and filled teeth.
● e. It is difficult to calibrate examiners.
● It is reversible.
● There is no gold standard

467 The biological process by which the architecture and function of the lost tissue is completely
restored".
In Periodontology this is a definition of which of the following
● New Attachment
● Regeneration
● e. Repair
● Primary Healing
● Reattachment
468 There has been much debate regarding the effectiveness of fluoride in water for preventing
tooth decay. A systematic review of the evidence drew conclusions as to the reductions in
decay which can be expected.
Fluoridation of public water supplies:
● Reduces tooth decay by 10%.
● Reduces tooth decay by 20%.
● e. Reduces tooth decay by 30%.
● Reduces tooth decay by 40%.
● Reduces tooth decay by 50%.

469 An obese 40 year old who takes metformin is seen at the end of a morning clinic. After
administration of local analgesia in the supine position the patient complains of feeling unwell.
She is pale and sweating and is confused. No pulse was detectable. She was given oxygen and
maintained in the supine with no improvement in condition.
What would be the most appropriate drug to administer next?
● Glucagon
● Glucose
● Adrenaline
● Hydrocortisone
● Chlorphenamine

470 A 12 year old boy is brought in by his mother who is concerned about the appearance of his
central and lateral maxillary incisors. She says they look like they might be decayed. They are a
new family to the practice having recently moved to the area from Birmingham. On examination
you notice a generalised whitening of the incisors which seems to worse at the incisal edges.
After you have dried the teeth you notice that the whiteness appears to be linear running from
mesial to distal. There is a similar pattern on the lower incisors.
What is the most likely explanation for the presentation of the boy's incisors?
● Tetracycline staining
● Early caries (white spots)
● Fluorosis
● Demarcated Enamel Opacities
E. Previous treatment with composites

471 A 23 year old patient attends complaining of pain in an upper right molar and is keen to
keep the tooth. The pain is typically sharp in nature, is triggered by cold and persists after
removal of the cold stimulus. The tooth is not tender to percussion; a
radiograph of the upper right first molar shows a large radiolucency extending to the pulp horn
but no peri-radicular changes.
What treatment is most likely needed in this case?
● Oral hygiene instruction and fluoride application
● Excavation of caries and placement of a permanent restoration
● Root-canal treatment
● Indirect pulp cap and restoration
● Direct pulp cap and restoration

472 A child of 5 years attends with pain from a grossly decayed lower right D which has a
discharging sinus. He is a hemophiliac.
Which of the following is the most appropriate treatment to relieve his pain?
● Antibiotics
● Extraction
● Fluoride application
● Non-vital pulpotomy
● Vital pulpotomy

473 You are designing a partial denture for a patient with several missing teeth in the maxilla.
The reason for surveying the model prior to designing the denture is to:
● Measure and mark out hard and soft tissue undercuts on the casts
● Relate the intended position of the inter-papillary plane of the patient to the casts
● Establish the position of the post dam
● Relate the maxillary and mandibular casts
● Aid setting up the prosthetic teeth prior to trial insertion

474 ​The parotid gland is one of the major salivary glands that supply the oral cavity.
Where i​s the orifice of the duct of the parotid gland located?
● At the hamular notch
● In proximity to the incisive papilla
● On the buccal mucosa near the maxillary second molar
● Slightly posterior to the mandibular central incisors
● Distal to the maxillary third molars on the palatal side

475 Any patient receiving treatment under IV sedation must have their blood pressure recorded
as part of their assessment. What is the maximum blood pressure that is generally regarded as
being compatible with safe sedation in general dental practice?
A.160/95
B.140/95
● 160/90
● 170/100
● E.120/80
476 A 13 year old boy with Down's syndrome attends for the first time. He is cooperative and
has no relevant medical history. He is caries free, apart from two small occlusal cavities in his
lower second primary molars. His mother requests that these teeth are restored.
Which of the following is the most appropriate management?
● Arranging to extract these teeth at a future visit.
● Arranging to place two simple restorations at a future visit.
● Reassurance that these teeth can be left until they exfoliate naturally, with no treatment
being necessary.
● Recommending use of fluoride mouthwash.
● Take OPT to check on unerupted teeth.

477 A 3 year old child attends, showing evidence of bottle caries and also decay affecting
primary molars. The child's mother has heard that some sugars are damaging her child's teeth.
She is not sure which foods to avoid.
From the following, which are you most likely to suggest she avoids?
● Bread
● Cheese
● Ground almonds
● Pure fruit juices
● Whole fruits and vegetables

478 An adult patient complains of her prominent upper front teeth and receding chin. She says
she did not get 'braces' when she was younger because the family had to move area a lot. Her
oral care and health is good, and she has a 9 mm overjet.
Where is the most suitable place to refer her? "
● A GDP friend who has a special interest in orthodontics and has been trained to use
fixed appliances.
● A specialist practitioner who uses a lot of functional appliances.
● A private specialist practitioner, because she's too old to get NHS treatment now.
● A hospital consultant, as it is likely she will need surgery now to correct her problem.
● The nearest dental hospital, although it is 70 miles away.

479 You are meeting a three year old patient for the first time. His mother explains that he had a
unilateral cleft lip and palate that was repaired in infancy. She has heard that orthodontics will
usually be required when he is older. She asks you what is the
commonest orthodontic problem that occurs with a repaired cleft palate. What.:.is the most
appropriate answer?
● A contracted maxillary arch -
● An anterior open bite.
● An elongated maxillary arch.
● An expanded maxillary arch.
● An increased overjet
480 Chronic periodontitis is a disease of the periodontium initiated and sustained by microbial
plaque. Which of the following is the main diagnostic feature of the disease?
● Tooth Mobility
● Loss of attachment
● Pain
● Gingival recession
● Drifting

481 Gingivitis and periodontitis are associated with the loss of normal tissue.
Which of the following is the principal cause of the tissue loss?
● Bacterial enzymes such as collagenase and hyaluronidase
● Release of lysosomal enzymes and oxygen free radicals by PMNs and macrophages
● Cytotoxic metabolic by products such as ammonia, hydrogen sulphide and toxic amides
● Complement activation
● Release of bacterial endotoxins and exotoxins.

482 Localised gingival recession is recession of the gingiva confined to one surface of the tooth.
Which of the following is the most important factor leading to the development of localised
gingival recession?
● Habit activity.
● Class 2 division 2 malocclusion with complete overbite
● Excessive toothbrushing
● Bony dehiscence
● Fenestration
483 A 48 year old woman complains of a sore area on the right buccal mucosa adjacent to a
restored tooth. The lesion has a lichenoid appearance and this is confirmed histopathologically
following a biopsy. Which of the following restorative materials is most frequently associated
with lichenoid changes.?
● Gold
● Amalgam
● Porcelain
● composite
● Glass ionomer cement

484 A patient attends your surgery complaining of severe pain, swelling and mobility associated
with a lower first molar tooth in which there is a broken filling. A periapical radiograph indicates
that the tooth has not been root filled and there is loss of apical lamina dura associated with the
distal root and at the bifurcation. The periodontal bone support is good. There is no significant
pocketing. What is the likely diagnosis?
● Chronic periapical periodontitis
● An acute periodontal-endodontic periodontitis
● A chronic periodontal-endodontic periodontitis
● An acute periapical periodontitis
● Para radicular periodontitis

485 An upper incisor in a 16 year old patient has suffered trauma and the coronal tissue has
been lost. The tooth has been endodontically treated. How will the teeth best restored?
● With a fibre post, direct core and crown.
● With a direct core and crown.
● With composite
● With an indirect post-core from a pre-fabricated pattern and a crown
● With a custom indirect post-core and crown

486 A 55 year old female presents at your surgery complaining of a sharp pain of no more than
a few minutes duration arising from her lower incisors. The pain only occurs when she eats or
drinks cold or sweet foods and only lasts for as long as the stimulus is present. A previous
dentist applied a varnish to the teeth affected which seemed to help. The teeth indicated by the
patient appear healthy but when you blow air onto them the patient experiences the pain.
What is the most likely cause of the patient's pain?
● Acute/reversible pulpitis
● Dentine sensitivity
● Chronic/irreversible pulpitis
● Atypical odontalgia
● Trigeminal neuralgia

The pain is sharp and sudden, in response to an external stimulus.​[7]​ The most common trigger is

cold,​[1]​ with 75% of people with hypersensitivity reporting pain upon application of a cold stimulus.​[5]

Other types of stimuli may also trigger pain in dentin hypersensitivity, including:

● Thermal – hot and cold drinks and foods,​[7]​ cold air, coolant water jet from a dental

instrument.

● Electrical – ​electric pulp testers​.[8]


● Mechanical–tactile – dental probe during dental examination,​[8]​ ​periodontal scaling and

root planing​,[8]​
​ toothbrushing.​[7]

● Osmotic – hypertonic solutions such as sugars.​[8]

● Evaporation – air blast from a dental instrument.​[8]

● Chemical – acids,​[8]​ e.g. dietary, gastric, acid etch during dental treatments.

The frequency and severity with which the pain occurs are variable.​[2]

487 As a newly qualified dentist you are offered a contract in which remuneration is on
capitation basis. What is the most important and well-recognised feature of this method of
remuneration?
● Tends to encourage under-prescribing
● Tends to encourage high technical quality of work
● Tends to encourage high output of procedures
● Tends to encourage over-prescribing
● Tends to encourage low technical quality of work

488 A 50 year old male patient has a Class III jaw relationship with an anterior open bite. It is
planned to restore his lower right second molar, which has suffered tooth wear and fracture,
with an indirect restoration. This tooth has approximately 2mm of coronal height. What would be
the most suitable approach to restore this tooth?
● Provide an adhesively retained gold onlay
● Provide a conventional full crown
● Increase the vertical dimension and provide a full crown
● Surgically crown lengthen and provide a gold crown
● Provide an adhesively retained ceramic onlay

Onlays are indicated where there has been a significant loss of occlusal tooth substance or
where caries or breakdown of a pre-existing large restoration has grossly weakened the
remaining tooth substance. The improved strength of the alloy under occlusal forces is the
important property.

489 Gracey curettes are specially designed for subgingival debridement and are site specific.
Which curette is recommended for use on the lingual surface of a lower second molar?
A.Gracey Curette number 1 and 2.
B.Gracey Currette number 3 and 4
C.Gracey Currette number 7 and 8.
D. Gracey Currette number 11 and 12.
E. Gracey Currette number 13 and 14.
490 Lignocaine (2%) is widely used in dental procedures. It is most often used in combination
with epinephrine (1 in 80,000).
In which one of the following patients is the use of epinephrine containing local analgesia
potentially hazardous?
● Patient with severe hypertension
● Patient on tricyclic antidepressants
● Patient who is an alcoholic
● Patient on monoamine oxidase inhibitors (MAOls)
● Patient with Grand Mal epilepsy
In patients with severe hypertension or unstable cardiac rhythm, the use of adrenaline
with a local anaesthetic may be hazardous. For these patients an anaesthetic without
adrenaline should be used.
http://www.evidence.nhs.uk/formulary/bnf/current/15-anaesthesia/152-local-anaesthesia
#PHP8665

491 An 80 year old male presents at your surgery complaining of a sharp stabbing pain of no
more than 2-3 minutes duration arising from his upper left pre-molar region. The pain can be
brought on by cold stimuli but also occurs spontaneously and has been sufficiently severe to
wake the patient from sleep. He has tried taking paracetemol but this has been of no benefit. On
examination the patient has a heavily restored upper left 4, which is vital to electrical pulp
testing and shows no radiographic evidence of caries. Blowing cold air onto the tooth produces
the pain but the pain also occurs spontaneously when you are examining the patient. There is
no evidence of a crack or fracture in the tooth itself.
What is the most likely cause of the patient's pain?
● Acute/reversible pulpitis
● Dentine sensitivity
● Chronic/irreversible pulpitis
● Atypical odontalgia
● Trigeminal neuralgia

492 A patient complains of a lower incisor which has been mobile for several months. The
radiograph indicates a normal level of bony support although the periodontal space has
widened. The apical bone appears normal. The tooth is tender to pressure.
Which of the following tests and or examinations would be most likely to provide a diagnosis?
● Masticatory muscle palpation
● Electric pulp test
● Occlusal examination >occlusal trauma
● Ethyl chloride test
● Hot gutta percha application

493 A nine year old boy presents with a class I occlusion with no crowding or overjet with a
grossly carious upper left 6 which is not suitable for restoration. The upper left 7 is very near to
eruption. Upper right 6, lower right 6 and lower left 6 are sound and fissure sealed. The oral
hygiene is good. What is the most appropriate extraction pattern for this patient?
● Extract all four 1st permanent molars
● Extract upper left 6 and lower left 6
● Extract upper left 6 and lower right 6
● Extract upper right 6 and upper left 6
● E. ​Extract upper left 6 only

494 A new patient attends your practice. At the initial examination, you carry out a Basic
Periodontal Examination (BPE).
What does a BPE score of 1 signify?
● Periodontal health
● Probing depths greater than 3.5mm
● Presence of overhanging restorations
● Presence of supragingival calculus
● Presence of bleeding on probing

495 A patient presents with a history of pain in the right pre-auricular region. There is an
intermittent click during opening; when the click is not present the patient can open to a normal
range. On examination the masticatory muscles, including the lateral
pterygoid, on the RHS side are tender. What is the most likely diagnosis?
● Bruxism
● Myofascial pain (or Pain dysfunction syndrome)
● Disc Displacement with Reduction
● Osteoarthrosis
● Disc Displacement without Reduction
496 A 23 year old male presents to your surgery. He lost his upper lateral incisors some 10
years ago in a swimming pool accident. Since then he has been wearing a 'spoon' denture
which he now feels in aesthetically unacceptable. He has sought an opinion on
dental implants but has been told that he would need bone grafting for this to be successful and
he is not prepared to undergo this. His dentition is excellent with no restorations and a Class I
occlusion. He wants some advice on what the best treatment might be.
Which option would you put first on your list of possibilities?
● Two fixed - fixed resin bonded bridges using the central and canine teeth
● Two cantilever resin bonded bridges from the canines.
● Two conventional fixed - fixed bridges from the canine
● Conventional cantilever bridges from the canines
● Cobalt chrome partial denture

497 All the films which have been manually processed by an unsupervised new trainee dental
nurse in your practice are extremely dark. Which of the following errors could lead to a dark
film?
● Insufficient exposure
● Films have been left in the developer for too long a period of time
● Films have been in the fixer for too long a period of time
● The concentration of the developer is too dilute
● The concentration of the fixer is too dilute

Q498: your colleague is having problem as every panoramic film that he produce shows
extremely wide anterior teeth which are blurred. The film also invariably has the images of the
condyles cut off from the sides of the film. The most likely cause of this?
● The patient is incorrectly positioned too far forward relative to the image layer
● The patient is incorrectly positioned with the chin too low
● The patient is incorrectly positioned too far back relative to the image layer
● D. The patient is in a slumped position in the machine
E. The patients is incorrectly positioned with the head tilted in the machine
OPG fault - patient too far back

Wide, blurred anterior teeth, loss of apices.Ghosting of rami, spread-out turbinates, ears, and nose in image,
condyles off lateral edges of film.When patient too far back- anterior wide teeth plus lateral side of condyle cut off.

When patient chin slumped down -condyle HEADS are cut off
http://www.slideshare.net/PARTHPMT/panoramictechique-errors

499 When an individual is exposed to radiation a certain amount of radiation is needed before
clinical signs of damage to somatic cells appear. For these effects to occur a minimum radiation
dose has to be exceeded and this is known as:
● Background radiation dose
● Threshold dose
● Equivalent dose
● Absorbed dose
● Effective dose

500 Dentists are encouraged to carry out a thorough soft tissue examination and be vigilant for
any lesions that might possibly be malignant or have malignant potential. Currently it is not
recommended that dentists use a mucosal staining method for screening
the general population for oral cancer. What is the most likely reason for this advice?
● Unnecessary intervention where there is a false positive test result
● False reassurance where there is a false negative test result
● Low uptake of screening by those with low risk
● Low uptake of screening by those with high risk
● Negative result reinforcing existing bad habits

S-ar putea să vă placă și